Family Med EOR

Ace your homework & exams now with Quizwiz!

A 30-year-old man, who has a 10-year history of alcohol overuse, presents with a symmetrical erythematous facial rash in areas that are exposed to the sun. He reports he has been a little confused and forgetful lately. A urine test shows a deficiency in N-methylnicotinamide. What additional symptom might be seen in this patient due to this deficiency? A) Diarrhea B) Flushing C) Gingivitis D) Paresthesias

A) Diarrhea

A woman with well-controlled, type 2 non-insulin dependent diabetes mellitus presents to the clinic with recurrent vaginal yeast infections. Which of the following medications is most likely contributing to her chief complaint? A) Empagliflozin B) Glipizide C) Linagliptin D) Metformin

A) Empagliflozin

A patient receiving quadruple drug therapy for active tuberculosis develops pain and paresthesias in his fingers and toes. What is the most likely agent responsible for these symptoms? A) Ethambutol B) Isoniazid C) Pyrazinamide D) Rifampin

B) Isoniazid (causes peripheral neuropathy, reason for co-treatment with B6)

An 86-year-old woman presents with recent onset of intractable headaches, jaw claudication, and visual field changes, including diplopia. Her past medical history is significant for Polymyalgia Rheumatica. Based on the most likely diagnosis, what is the primary reason for prompt diagnosis and treatment of this condition?

Prevent blindness (classic presentation of Giant Cell Arteritis)

A 19-year-old man presents with pain and deformity of his right dominant shoulder after a sudden jerking movement to the same from a wrestling competitor approximately 1 hour ago. He states he felt a clunking sensation when it happened. He was unable to continue wrestling and has pain with movement of the right shoulder. What diagnostic studies should be performed?

Anterior/Posterior (AP) and axillay or transscapular lateral (Y-scapula) radiographs

What is the treatment of choice for severe pelvic inflammatory disease in a patient with no known drug allergies? A) Ampicillin-sulbactam 3 g IV every six hours B) Cefoxitin 2 g IV every six hours plus doxycycline 100 mg orally every twelve hours C) Ceftriaxone 250 mg IM plus azithromycin 1 g orally one time D) Ciprofloxacin 500 mg orally every twelve hours

B) Cefoxitin 2 g IV every six hours plus doxycycline 100 mg orally every twelve hours

A 58-year-old woman presents to the clinic with right facial weakness since this morning. Which additional finding would suggest a diagnosis of cerebrovascular accident rather than Bell's palsy? A) Drooping of the right side of the mouth when smiling B) Inability of the right eye to abduct C) Inability to fully close the right eyelid D) No wrinkling of the right side of the forehead

B) Inability of the right eye to abduct

A patient comes to see you for a growth on his forearm. He has had it for 7 years and is concerned that it may be enlarging. You observe a well circumscribed, pink, slightly pigmented, raised nodule about 1 centimeter in diameter. The center appears slightly ulcerated. What is the most likely diagnosis?

Basal Cell Carcinoma -usually starts as small indented nodule localized around hair follicles on sun exposed regions -ulcerates as enlarges -locally invasive, rarely metastasizes

Biliary Decompression for Cholangitis

1. ERCP 2. Percutaneous transhepatic drain 3. surgery

PPS23 Vaccine Timing

>65 years Immunocompromised

Clinical Presentation of Diverticulitis

Abdominal pain (MC LLQ) Nausea/vomiting Fever Changes in stool

A 33-year-old man presents with shortness of breath, wheezing, mild fever, and fatigue. He has had several similar episodes in the past, and each previous episode began after a cold that moved into his chest. Over the past several weeks, he has had a productive cough most mornings. He smokes on a social basis. What is the most likely diagnosis?

Acute asthmatic bronchitis

A 35-year-old man with known stage 3 chronic kidney disease due to diabetes presents with fatigue. His blood sugars are acceptable; his mood is euthymic, and he is sleeping and eating well. He does not smoke, and he has no known toxic environmental exposures. Work up shows normochromic normocytic anemia, with a hemoglobin of 11 g/dl, which is decreased from 12 g/dl 6 months ago. Medications include lisinopril, furosemide, and insulin. Iron 50 ug/dl (normal 50-150 ug/dl) Ferritin 200 ng/ml Ratio soluble transferrin receptor/log ferritin: Reduced Reticulocytes 0.25% Platelets: Normal Total white blood cell count 5x103/mm3 B12 levels, serum: Normal What is the most likely diagnosis?

Anemia of chronic disease

A 70-year-old male presents to your clinic complaining of an acute onset of severe periumbilical pain with nausea and vomiting. Past medical history is remarkable for coronary artery disease with atrial fibrillation. Physical examination is remarkable for minimal abdominal distension and guaiac positive stool. The suspected diagnosis is acute mesenteric ischemia. Which is the gold standard diagnostic imaging study to confirm the suspected diagnosis?

Angiography

Influenza Vaccine Timing

Annually after 6 months

A 7-year-old boy presents with a 1-week history of wheezing and dyspnea on any exertion (with productive cough). On physical examination, bilateral rhonchi are heard. After a few days of treatment, spirometry is done on the patient. The findings are shown in the table. Total lung capacity (TLC) is 111% on spirometry. What is the most likely diagnosis?

Asthma

Medications all CHF patients get

BP management Beta blockers

Medications all CAD patients get

Beta blocker Aspirin (lifetime) Plavix (12 months) Statin

Hepatitis B Vaccine Timing

Birth 1-2 months 6-18 months

Diabetes Screening

Blood glucose (fasting >126, random >200) A1c (>6.5%, >7.5% if >65, >8% if >75) Urine glucose

Labs for COPD

CBC- anemia Elevated serum bicarb AAT deficiency

Lifestyle Treatment of Osteoporosis

Calcium/vitamin D Diet (gluten free?) Exercise Intensity of exercise Cessation of smoking

HPV vaccine

Can start at 9 years but recommended at 11-12 Max age: 26 2 doses if started before 14, 3 doses if started after

DKA

Characterized by ketoacidosis and hyperclycemia

Renyold's Pentad

Charcot's Triad Hypotension and AMS

Treatment of PBC

Cholestryramine Ursodeoxycholic acid Liver transplant

Diagnosis of constipation

Choose 2: -straining -lumpy, hard stool -feeling of incomplete emptying -use of digital maneuvers -sensation of blockage or obstruction (25%) -decreased frequency >3 months Rarely have loos stools w/o laxatives No IBS diagnosis

A 53-year-old woman presents with a 1-year history of chronic cough. The cough produces large volumes of grossly purulent sputum. She has a history of recurrent respiratory infections; they resulted in 5 hospitalizations in the past year. She also had similar complaints during the previous year.. Shortness of breath limits her daily activity considerably. Upon pulmonary examination, bilateral breath sounds are audible, with inspiratory and expiratory crackles at the lung bases. Chest X-rays reveal increased lung volumes, flattened diaphragm, and tram track lines. What is the initial diagnosis?

Chronic bronchitis

Chronic Cholecystitis

Chronic inflammation due to repeated episodes of acute attacks causing mucosal atrophy and fibrosis of the gallbladder wall

Primary Sclerosing Cholangitis

Chronic thickening of the bile duct walls leading to cholestatic liver disease

A 12-year-old boy presents with left shoulder pain and deformity after a fall from his skateboard while attempting a half-pipe maneuver an hour ago. He heard a "cracking" sound and was unable to continue skateboarding. He has pain with movement of the arm in any direction. On exam, you note tenderness over the superior central shoulder with a palpable deformity and "tenting" of the skin. What is his most likely diagnosis?

Clavicle fracture

Diagnosis of GERD

Clinical

Diagnosis of UTI

Clinical UA- pyuria Urine culture- E. coli MC CT or US (if suspected blockage)

A recognized complication of sleep apnea is

Congestive heart failure (CHF)

A white female comes into the evening clinic with a complaint of red eyes with a sticky discharge. Her eye feels irritated, the pain is mild. She tells you she shared towels with her boyfriend who had a red eye. She asks you if you think her boyfriend gave her an infection. What is your working diagnosis?

Conjunctivitis

Repeat upper EGD for PUD if

Duodenal ulcer (higher risk of malignancy) Symptoms persist despite therapy Unclear etiology Giant ulcer Bleeding High suspicion of malignancy Risk factors for gastric cancer

Classic S/S of Asthma

Dyspnea Cough Wheezing *episodic with triggers*

A 29-year-old woman, G0P0, and her husband present to your office after 1 year of infertility. Their histories elicit irregular menses in the woman and that the man is an avid cyclist. Apart from semen analysis in the man, what laboratory and/or diagnostic studies should be pursued in the woman to further evaluate this couple's infertility?

Fasting prolactin, TSH, FSH, LH, hysterosalpinogram

A 23-year-old woman presents with an anaphylactic reaction after being stung by several bees. She complains of wheezing and shortness of breath. On examination, the client is in acute distress. BP is 98/56 mmHg, PR 110/min, RR 28/min, and temperature 98.7°F. She is immediately treated with supplemental oxygen. In treating this condition further, what drug is required most urgently?

Epinephrine

A 30-year-old man presents with a 2-month history of coughing and a 2-day history of coughing blood. He has been losing weight and sweating at night. On physical examination, the patient appears wasted, tachypneic, has bronchial breath sounds on the right upper lobe, and crepitations on the left upper lobe and right mid zone. His direct sputum result comes back positive for acid-fast bacilli with Ziehl-Neelsen stain. His sputum is sent for culture, and treatment is started. Refer to the image. Retrobulbar neuritis is the predominant adverse effect of what drug?

Ethambutol INH: Hepatotoxicity, peripheral neuritis, cutaneous hypersensitivity, rarely can cause optic neuritis RMP: Hepatotoxicity, nephrotoxicity, red discoloration of the body fluids, 'Flu-syndrome,' and thrombocytopenic purpura PZA: Hepatotoxicity, hyperuricemia ETH: Retrobulbar neuritis STM: Nephrotoxicity, ototoxicity

A white male comes into the ER with the complaint of pain in his right eye. He was whittling and felt like something went into his eye. This happened four hours ago. He has washed his eye with water, but it still feels funny. What is your working diagnosis?

Foreign body

A 63-year-old Hispanic-American man presents with a 3-month history of chronic intermittent diarrhea with 5 - 6 watery stools per day; some are tinged with blood. During this period, he has had progressive fatigue and intolerance to exercise; he has lost 15 pounds (6% of body weight).Vital signs are within normal ranges. On physical examination, there is noticeable pallor. Heart sounds are regular and of normal frequency, both lung fields are clear to auscultation, the abdomen is soft and diffusely tender to palpation, particularly in the right quadrants. Rectal examination is painless, and an enlarged prostate is palpated, but there are no masses or blood. Anoscopy shows grade I hemorrhoids. There is no peripheral edema.A complete blood count shows the following: Ht 30% Hb 9.4 g/dL MCV 77 fl MCHC: 25 g/dL WBC 8.600 /mm3. w/ normal differential, platelets 460.000 /mm3; iron 56 µg/dL ferritin 5.5 µg/dL. What is the most appropriate next step in management?

Full colonoscocpy

Transferrin saturation would be increased with

Hemolytic, megaloblastic, sideroblastic anemia Hemochromatosis or other iron overload disorders

Risk Factors of Hypothyroidism

Goiter Hx of autoimmune disease Previous radioactive iodine therapy Head and neck irradiation Family Hx Certain medications

A 25-year-old woman presents with inability to achieve intercourse with her partner since their relationship began 6 months prior. She states that she has been sexually active for 5 years, and penetration had been possible until recently; now, she cannot even insert a tampon due to the pain. Further history reveals that a year ago, her partner at that time forcefully made her have intercourse on several occasions. What is the next step in the workup of this patient to confirm the suspected diagnosis?

Gynecologic examination

A 50-year-old man presents with a 3-month history of weakness, fatigue, and abdominal discomfort. Upon further questioning, he acknowledges a lack of sexual desire. He denies any photosensitivity. On physical examination, his liver is enlarged, and his spleen is palpable. He has abnormal skin pigmentation on his face, neck and his elbows and which gives his skin a metallic gray hue. His laboratory results are in thechart. TEST RESULTS REFERENCE RANGE TIBC 275 μg/dL (250 - 350 μg/dL) Plasma iron 220 μg/dL (80 - 160 μ/dL) Transferring saturation 90% (16 - 57%) What serious complication is associated with the patient's condition?

Hepatocellular carcinoma Pt's S/S consistent with hemochromatosis.

A 42-year-old hypertensive, diabetic Native American woman is scheduled for a follow-up visit today. She says that her father recently passed away after having a heart attack. She is extremely worried and wants to know what she can do to reduce her risk of cardiovascular disease (CAD). You explain to her that that the risk factors for CAD are classified as either non-modifiable or modifiable. What is the strongest non-modifiable risk factor for CAD in this woman?

Her ethnicity

Diagnostics of Cholecystitis

High WBC +/- LFT or pancreatic enzyme elevation US (DOC) HIDA CT (r/o complications)

Diagnostics of Cholangitis

High WBC with left shit LFTs- very high alk phos and bilirubin, high ALT CMP- elevated BUN and creatinine Blood culture ERCP (TOC and DOC) US

A 25-year-old woman presents to the ER after a syncopal episode. She had loss of consciousness 3 times over the past 12 months. Each event occurred during or just after physical exercise. On PE: BP 110/70 mm Hg, HR 75/min, normal S1/S2, and a III/VI systolic ejection murmur is heard best at the left sternal border that decreases with squatting. The EKG shows a normal sinus rhythm with diffuse increased QRS voltage. What is the most likely diagnosis?

Hypertrophic Cardiomyopathy

Causes of Osteoporosis

Hypogonadism Drugs Smoking IBD Celiac's disease Cystic fibrosis Hx or current hyperthyroidism Hypercalciuria Hyperparathyroidism Depression Abnormal pregnancy and lactation Genetics

A 29-year-old woman inquires when she should have her first mammogram. Her family history is significant for the death of her mother and grandmother due to breast cancer at the age of 32 years. Presently, the woman is asymptomatic. What is the recommendation for mammography for this patient? If you have a strong family history, the National Comprehensive Cancer Network recommends, starting at age 30, you get a: Clinical breast exam every 6-12 months Mammogram every year Breast MRI every year

Immediately -based on recommendation and early genetic burden

Subsequent Treatment of GERD

Increase H2RA Add PPI Alginates Reflux inhibitors Surgery

Diagnostics of Pancreatitis

Increased amylase and lipase CBC- leukocytosis, decreased Hct, eosinophils CMP- elevated glucose, alk phos, and ALT Abdominal CT (no US)

Common Precipitating Factors of DKA

Infection and inadequate insulin therapy MC New onset Type 1 DM Cocaine

Cholangitis

Infection of the biliary tree (MC cause: obstruction of common bile duct) *MC organism: E. coli, Enterococcus, Klebsiella, Enterobacter*

Acute Cholecystitis

Inflammation due to complete cystic duct obstruction and/or bile stasis

Clinical Presentation of HHS

Insidious onset Polyuria Polydipsia Weight loss Neurologic symptoms Signs of dehydration Neurologic findings

Clinical Presentation of PSC

Jaundice and pruritis (MC) Fatigue Weight loss RUQ pain +/- HM and/or SM

A 13-year-old girl presents for her school physical. On examination, you notice the posterior curvature of her thoracic spine to be very prominent and bulging backward. What type of deformity of the spine does she have?

Kyphosis

Labs for SBO

Leukocytosis with left shift Anemia +/- electrolyte imbalance Metabolic acidosis Serum lactate (ischemia) Blood culture

Treatment of Hypothyroidism

Levothyroxine

Risk Factors for Osteoporosis

Low BMD Advanced age Previous fracture Long-term glucocorticoid therapy Low body weight Hx of hip fracture Cigarette smoking Excess alcohol intake Race/ethnicity

A 55-year-old woman comes to your clinic presenting with episodic vertigo, tinnitus, hearing loss, and ear fullness. Her ear and eye physical examination are unremarkable. You perform a Dix-Hallpike maneuver which is negative. There are no carotid bruits noted on auscultation. Which of the following is the best initial treatment for this patient?

Low sodium diet

A 73-year-old man presents with the inability to actively raise his left non-dominant arm to retrieve plates from the kitchen cabinet. This began a month ago after his shoulder pain improved. He had a history of pain in that shoulder for over 6 months that kept him from sleeping on the left side and the pain would wake him often. There was no specific injury he can recall although he felt a pop a month ago while taking out the trash. What is the diagnostic study of choice if surgery is indicated?

MRI (high suspicion for torn rotator cuff, MRI necessary for information needed prior to surgical repair)

Total Iron Binding Capacity (TIBC)

Measure of all proteins available for binding mobile iron *increased in 70% of iron deficient patients*

Medications for DM

Metformin (1st line) Statin BP control

A 45-year-old woman with a 25-year history of type I diabetes presents for follow-up of her diabetes. Her spot albumin/creatinine ratio was 100 mg/g 4 months ago, and it was confirmed at 100 mg/g yesterday. Her urinary analysis shows no cells, casts, or blood. Her creatinine is 0.7mg/dl, and her estimated glomerular filtration rate is 95 ml/min/1.73m2. How would her proteinuria be described?

Microalbuminuria Spot Urine Albumin: Normal: <10mg Microalbuminuria: 20-200mg Macroalbuminuria: >200mg Nephrotic range albuminuria: >3000mg

Labs for Anemia

Microcytic: iron studies Normocytic: reticulocyte count Megaloblastic: B12, Folate, Homocysteine level, serum methylmalonic acid

COPD Group C

Minimally symptomatic, high risk of exacerbation Tx: LAMA with SABA prn

COPD Group A

Minimally symptomatic, low risk of exacerbation Tx: SABA or SAMA prn

COPD Group D

More symptomatic, high risk of exacerbation TX: LAMA-LABA or LABA-ICS

COPD Group B

More symptomatic, low risk of exacerbation Tx: daily LABA or LAMA with SABA or SAMA-SABA

A 42-year-old woman works full-time as a data entry clerk and often puts in many hours of overtime. She has started to notice numbness and tingling in her right thumb, index finger, middle finger, and half of her ring finger; symptoms are especially severe at night The numbness and tingling were intermittent for months, but they have become persistent during the past few days. What could be considered a predisposing factor for this patient's condition?

Myxedma -metabolic predisposing causes for carpal tunnel syndrome

Treatment of Pancreatitis

NPO Avoid triggers IV fluids Pain/nausea control IV antibiotics Surgery

Treatment of Cholecystitis

NPO IV hydration Antibiotics Analgesia Cholecystectomy

Coombs' test

Negative is evidence for hemolysis from an immune etiology

Transferrin

Negative-acute phase reactant protein

A 44-year-old man presents for follow-up of poorly controlled type I diabetes mellitus, which was diagnosed 32 years ago. What change on his funduscopic examination would indicate a need for urgent referral to an ophthalmologist?

Neovascularization (hallmark of proliferative diabetic retinopathy)

Alarm Symptoms of GERD

New onset >60 Evidence of bleeding Iron-deficient anemia Anorexia Unexplained weight loss Dysphagia Odynophagia Persistent vomiting Fam Hx of GI cancer

Treatment for MDR UTI

Nitrofurantoin Fosfomycin Pivmecillinam

A 51-year-old man presents with a 3-day history of severe pain associated with bowel movements. The pain lasts up to 1 hour following each bowel movement. His stools are described as "hard", but they retain their normal brown color, with the occasional presence of a few drops of bright red blood. His dietary history is remarkable for a low-fiber diet lacking in raw fruits and vegetables. He denies any sexual contact, drug use, or other gastrointestinal symptoms. He takes ferrous sulfate for anemia and oxycodone for chronic joint pain. His rectal exam reveals a small tear in the lining of the anus. What is the most appropriate diagnostic test to order at this time?

No diagnostic tests are necessary -diagnosis of anal fissure can be made clinically

Outpatient Management of Diverticulitis

Oral abx (Fluoroquinolone + Flagyl) Diet changes Re-imaging (if no improvement)

Inpatient Criteria for Diverticulitis

Presence of complications Sepsis Microperforation High fever Significant leukocytosis Severe pain Age >70 Significant co-morbidities NPO Noncompliance Failed outpatient treatment

Clinical Presentation of Choledocholithiasis

Same as cholelithiasis but jaundice is MC

Labs for Folic Acid Deficiency

Serum folate <2 ng/mL Increased homocysteine level Normal serum MMA

Treatment of PSC

Stent Ursodiol

Choledocholithiasis

Stone in the common bile duct

Cholelithiasis

Stones inside the gallbladder

When to initiate pharmacological treatment of Osteoporosis

T scores between -1.0 and -2/5

A 62-year-old man with a past medical history significant for a 15-year history of hypertension presents with severe tearing chest pain radiating through to the back. His blood pressure is 180/110 mm Hg, heart rate is 120 BPM, and respiratory rate is 34/min. Physical exam findings include neck negative for bruits/jugular venous distension (JVD), lungs clear to auscultation, heart regular rhythm despite tachycardia, normal S1/S2 with an S4 present, and a grade III/IV diastolic rumbling murmur noted with patient leaning forward. Radial pulses are 1+ on right and 3+ on left. EKG reveals a sinus tachycardia and evidence of left ventricular hypertrophy. What is the most likely etiology of this patient's symptoms

Thoracic aortic dissection

Transferrin saturation

Transferrin and other mobile iron-binding proteins saturated with iron <15% in patients with iron deficiency

Diagnostic options for GERD

Upper endoscopy Esophageal impedance pH testing Esophageal manometry

A 25-year-old man presents to you with an acute otitis media with serous otitis in the right ear. You perform the Weber and Rinne tests. Which of the following results would you most likely expect to find?

Weber- sound is heard louder in right ear, Rinne- bone conduction exceeds air conduction in right ear Conductive hearing loss: Weber- louder in affected ear, Rinne- bone conduction > air conduction in affected ear Sensorineural hearing loss: Weber- louder in unaffected ear, Rinne- air conduction > bone conduction in affected ear

A 3-year-old girl presents with a 2-day history of sore throat and fever. This morning, she was hoarse and seemed to be having more difficulty breathing. On exam, she appears to be in distress and has an oral temperature of 100.0 F. Tympanic membranes are pink but not bulging. Nares are patent without rhinorrhea. She has a barking cough, stridor at rest, and nasal flaring. What treatment is most appropriate in the care of this child?

Admit patient, start humidified oxygen and intermittent racemic epinephrine

Medications all stroke patients get

BP management Treat the cause

Which of the following is most suggestive of delirium? A) Increased confusion in the evening time B) Progressive short-term memory loss C) Rapid decline in mental status over two days D) Sudden onset of slurred speech and facial droop

C) Rapid decline in mental status over two days

History to take for Constipation

Diet Medications Medical history Family history (CA) Last colonoscopy Assocaited symptoms

A 58-year-old man with a history of alcoholism presents with a 3-week history of dyspnea and fatigue. He has not seen a medical provider in multiple years and has no known medical diagnoses. He takes no daily medications. Work up reveals a normal troponin, elevated BNP, lateral T wave inversion on EKG, and evidence of cardiomegaly on CXR. You order a stat echo, which shows enlarged left ventricle and systolic heart failure evidenced by decreased contractility. What is the most likely cause of the patient's dyspnea and fatigue?

Dilated cardiomyopathy

A 3-year-old boy has velvety lax skin, hyperextensible joints, and mitral valve prolapse. What is the most likely diagnosis?

Ehlers-Danlos syndrome

Ferritin

Marker of stored iron Decreased: iron deficient anemia Increased: iron excess and/or inflammation and infection

Reticulocyte count is normal or increased for

Marrow hypoplasia, leukemia, infiltration

Severe Persistent Asthma

Persistent symptoms Waking up every night Medication needed multiple times a day Activity severely limited

Deficiency of what factor may predispose a person to recurrent thrombosis?

Protein C deficiency

A 2 week old female infant is seen for her newborn well baby exam after a normal birth and delivery. She has been nursing well, has regained her birthweight and her development appears normal for her age so far. Physical examination is normal with the exception that ophthalmoscopic evaluation reveals a faint white reflex in her right eye. What is the most likely diagnosis?

Retinoblastoma

A 25-year-old woman presents with fatigue. She has been diabetic for 10 years, and she takes multiple injections of insulin a day. She has no other medical problems. She takes no other medications, and she is not sexually active. Her skin shows diffuse scratch marks. What is the most likely diagnosis?

Secondary hyperparathyroidism (due to stage 3 kidney disease causing low calcium)

Imaging for SBO

Standing x-ray CT MRI (if pregnant)

Primary Biliary Cirrhosis

T-cell attack on bile ducts that leads to gradual bile duct loss and scarring, ultimately causing cirrhosis

The mother of a 5-year-old girl walks into her daughter's bedroom without knocking and discovers the girl stimulating her genitals. The girl's parents are concerned, but seem to be receptive to advice. What is the best response a physician could give?

"What disturbs you about this behavior?"

A 40-year-old man presents with atrial flutter with 2:1 atrioventricular (AV) conduction, giving him a pulse of 150 per minute, which is perfectly regular. He takes no medications regularly. You plan to provide him with urgent direct current cardioversion with conscious sedation. What should be used as the initial energy for cardioversion in order to restore sinus rhythm in patients with atrial flutter?

100 J While atrial flutter is the easiest to convert, if the Joules are too low and the cardioversion is ineffective, an increase of catecholamines can increase risk of v fib and cardiac arrest.

Meningococcal Vaccine Timing

11-12 years 15-16 years

Tdap Vaccine Timing

11-12 years Repeat every 10 years

MMR Vaccine Timing

12-15 months 4-6 years *4 weeks between doses, max age of 12 years*

Hepatitis A Vaccine Timing

12-23 months, 2 doses at least 6 months apart

Treatment of UTI

1st line: Nitrofurantoin Other: Bactrim, Fluroquinolones, 3rd/4th generation Cephalosporin Phenazopyridine (analgesic)

Medications for Hyperlipidemia

1st line: Statin Others: Fibrates*, Niacin, Omega-3 fatty acids, Ezetimibe, herbal remedies

Rotatvirus Vaccine Timing

2 months 4 months

Hib Vaccine Timing

2 months 4 months 15 months

DTaP Vaccine Timing

2 months 4 months 6 months 15-18 months 4-6 year (no 5th dose if 4th is after 4 years)

Inactivated Poliovirus Vaccine Timing

2 months 4 months 6-18 months 4-6 years

PCV12 Vaccine Timing

2 months 4 months 6 months 12-15 months

A 30-year-old HIV-positive man presents to your clinic for the first time with a history of pneumocystosis, and he is on trimethoprim/sulfamethoxazole for secondary prophylaxis. Primary prophylaxis of Pneumocystis pneumonia is recommended for patients at what CD4 cell count?

200 cells/mcL CD4 <50 cells/mcL: primary prophylaxis for mycobacterium avium complex

What is the therapeutic window for using recombinant tissue plasminogen activator in acute ischemic stroke?

3 hours

A 34-year-old gravida 3, para 3 woman presents with complaints of vaginal pressure and feeling as though something is "falling out of the vagina". The patient is three months postpartum. She notes recent onset of urinary incontinence with exercise. On exam, when the patient is asked to strain, a palpable smooth mass is noted involving the anterior vaginal wall. What is the most likely diagnosis? A) Cystocele B) Enterocele C) Rectocele D) Uterine prolapse

A) Cystocele

How would a hernia of the anterior vaginal wall resulting in the bladder descending to a lower anatomical position be documented in a medical chart? A) Cystocele B) Enterocele C) Rectocele D) Uterine procidentia

A) Cystocele

Which of the following laboratory results are most consistent with a diagnosis of immune thrombocytopenia? A) Isolated thrombocytopenia with platelets of predominantly normal morphology B) Isolated thrombocytopenia with platelets that lack granules C) Isolated thrombocytopenia with predominantly large platelets D) Thrombocytopenia with anemia and leukopenia

A) Isolated thrombocytopenia with platelets of predominantly normal morphology

Which of the following is the most reliable physical exam finding in the diagnosis of acute appendicitis? A) McBurney's point tenderness B) Murphy's sign C) Obturator sign D) Psoas sign

A) McBurney's point tenderness

Provider role in DM

A1c 2-4 times a year Annual: foot/eye exam, urine albumin, CAD Vaccines: flu annual, one-time PPS23 (+/- Hep B, TDaP, Shingles)

Clinical Presentation of SBO

Acute onset of severe abdominal cramping Nausea/vomiting Obstipation Signs of dehydration Abdominal distention Changes in bowel sounds

A 54-year-old male has had long term GERD symptoms. He has been on proton pump inhibitors and has had fair control of his symptoms. Other past history is unremarkable. He is a nonsmoker and drinks socially. Family history is significant for hypercholesterolemia in his father. Physical examination is unexceptional. An endoscopy a few years ago, revealed Barrett's esophagus by biopsy of the esophageal mucosa. He was recommended to have follow-up endoscopy every 2-3 years with mucosal biopsy. This screening was recommended to him because he is in danger of developing

Adenocarcinoma of esophagus

A 33-year-old woman presents after being found unresponsive in the bedroom of her home. She has a past medical history of depression, and her mother found an empty bottle of amitriptyline by her bedside. Otherwise, she has no other medical or surgical history. The patient is a non-smoker and does not drink alcohol.On physical exam, her pulse is 138/minute, blood pressure is 80/60 mm Hg, temperature is 101.2°F (38.4°C), and respirations 6/minute. Her heart sounds are normal and she has thready pulses. Her breath sounds are normal, but with shallow effort. The abdomen is soft and nontender. Neurologically, she moves her limbs from painful stimuli. Her skin is flushed with no needle marks. Her chest X-ray is normal, and the electrocardiogram demonstrates a wide complex tachycardia without ectopy. The patient is intubated and hyperventilated. What is the next appropriate step of the patient's management?

Administer sodium bicarbonate (TOC for TCA toxicity)

Management of SBO

Admit + surgical consult IV fluids NPO GI decompression with NG tube +/- antibiotics Surgery Barium swallow

Who to screen for diabetes

Adults 40-70 who are overweight or obese

Who to screen for hypothyroidism

Adults with increased risk Substantial hyperlipidemia Hyponatremia High serum muscle enzymes Macrocytic anemia Pericardial or pleural effusions Pituitary or hypothalamic disorders

Who to screen for Depression

All adult patients, including pregnant women Children >12 years

Who to screen for HTN

All adults annually Semiannually with risk factors or previous systolic BP of 120-129

A 61-year-old male presents with a recent history of increased fatigue with mildly increased exertional dyspnea. Patient denies any significant past medical history but states that he had some heart problems as a child, though he was never clear as to what was the problem. On cardiac examination, you hear an early diastolic, soft decrescendo murmur with a high pitch quality, especially when patient is sitting and leaning forward. No thrill is felt. Based on this patient's presentation, you expect the patient to have

Aortic regurgitation

A 62-year-old woman with a long-standing history of hypertension presents with a severe headache; it started this morning and is rapidly worsening. During the interview, she suddenly collapses. Your brief examination shows that she responds with extensor posturing on external stimuli. Her deep tendon reflexes are 3, and you elicit Babinski bilaterally. You also notice that her breathing has a peculiar pattern: deep inspiration with a pause at full inspiration, followed by a brief insufficient release and the end-inspiration pause. How do you best describe her respiratory pattern?

Apneusis (deep, gasping aspiration with a pause at full inspiration followed by a brief, insufficient release and the end-inspiration pause before expiration)

Patient Education for Asthma

Asthma and S/S Triggers Long-term vs short-term control Correct inhaler technique Change clothes when getting home Change sheets regularly Air out sheets

Clinical Presentation of PBC

Asymptomatic (MC) Fatigue Pruritis Jaundice Features of cirrhosis Xanthelasma Xanthoma Steatorrhea

Clinical Presentation of UTI

Asymptomatic (especially in elderly, pregnant, disabled, men) Dysuria Urinary frequency Urinary urgency Suprapubic pain Hematuria

Treatment of Cholelithiasis

Asymptomatic- nothing, elective surgery Cholecystectomy

A 64-year-old woman with a past medical history of hyperthyroidism presents with new-onset palpitations. The patient states that she has intermittent palpitations, and she is worried about heart disease. She is currently asymptomatic and reports no chest pain or shortness of breath. An EKG is obtained. Vital signs reveal a T 98.6° F, BP 134/88 mm Hg, P 119 beats/min R 12/min. The pulse is noted to be irregular. The EKG is shown here (irregularly, irregular rhythm). What arrhythmia does this patient most likely have?

Atrial fibrillation

Which of the following diseases is most commonly associated with a previous history of a hematologic disorder, such as myelodysplastic syndrome, aplastic anemia, or polycythemia vera? A) Acute lymphoblastic leukemia B) Acute myelogenous leukemia C) Chronic lymphocytic leukemia D) Chronic myelogenous leukemia

B) Acute myelogenous leukemia

A four-year-old boy with autism presents to the clinic with his mother with bilateral ear pain and decreased hearing. His mother states he was with his step-father for the weekend, and when he returned, he was inconsolable and complaining of bilateral ear pain. She states his stepfather said he "just started crying all of a sudden." Physical exam reveals bilateral auricular hematomas, as well as bilateral tympanic membrane perforation. There is also circumferential ecchymosis on his arms. Hearing tests reveal mild conductive hearing loss bilaterally. Computed tomography of the head without contrast is negative. What is the most appropriate next step? A) Audiology referral B) Child protective services consult C) Magnetic resonance imaging of the brain with and without contrast D) Ofloxacin 0.3% otic drops

B) Child protective services consult -MRI unnecessary if negative CT -Audiology and otic drops acceptable for outpatient treatment

Which of the following drugs slows atrioventricular nodal conduction and should be avoided in a patient with Mobitz type 2 second degree heart block? A) Atropine B) Digoxin C) Lisinopril D) Nitroglycerin

B) Digoxin -Nitro also not recommended but due to hypotension as it has no effect on conduction

Which of the following diagnostic studies would be most helpful to obtain in a patient complaining of syncopal episodes? A) Alkaline phosphatase B) Fasting glucose C) Sedimentation rate D) Thyroid stimulating hormone

B) Fasting glucose

During a routine health maintenance check of a 66-year-old man, you note the lesions seen above on the patient's skin. Which of the following is the most appropriate management? A) Cryotherapy B) No treatment, reassurance only C) Referral to dermatology for Mohs procedure D) Salicylic acid over the counter

B) No treatment, reassurance only

HTN Diagnosis

BP >140/90 on 2 occasions (>130/>80 if DM or CAD) Ambulatory BP mean >130/>80 BP >180/>120 BP >160/>100 with end-organ damage

A 26-year-old man residing in Thailand presents with high-grade fever, dull, frontal headache, malaise, anorexia, and vague abdominal discomfort of 7 days duration. He had mild diarrhea, dry cough, and myalgia. On examination, his temperature was 39 degrees C. His pulse was 65 per minute. He had a coated tongue, tender abdomen, and a soft palpable spleen. Clinical diagnosis was Enteric fever. Appropriate clinical samples were sent for culture and serology. What laboratory data would be helpful in making a definitive diagnosis?

Blood culture positive for S. typhi (definitive diagnosis for Enteric fever)

A 70-year-old man is brought into the office by his adult children because he has been leaving his house and wandering in the street. For the last six months, he became increasingly forgetful of names of family members and of places he often visited. He lost interest in his usual hobbies and is more restless. Which of the following is considered the first-line therapy for patients with mild-to-moderate Alzheimer's disease? A) Antidepressants B) Antiparkinsonian agents C) Cholinesterase inhibitors D) NMDA antagonists

C) Cholinesterase inhibitors

A 30-year-old semi-professional tennis player returns to your clinic for persistent right lateral epicondylitis. When he was initially seen a few weeks ago you prescribed conservative therapy with rest, ice, a compression band, and over-the-counter nonsteroidal anti-inflammatories as needed, but he has seen no improvement. What is your next most likely treatment option? A) Apply a long-arm cast B) Continue conservative therapy for another month C) Corticosteroid injection at the site of pain D) Refer to orthopedics for possible surgery

C) Corticosteroid injection at the site of pain

A 32-year-old man presents with persistent ringing in his ears that occurs daily and lasts throughout the day. This has been going on for several months, but recently has been getting gradually worse. He is a construction worker by trade. Health history is unremarkable except for lower back pain for which he pops some ibuprofen throughout the day. He denies any other medications. He does not smoke but usually has five to six beers on the weekend. His examination is unremarkable. What is the most likely cause of his tinnitus? A) Excessive alcohol consumption B) Excessive caffeine use C) Excessive ibuprofen use D) Exposure to excessive noise at work

C) Excessive ibuprofen use (NSAID use MC cause of tinnitus in young men)

A 68-year-old woman presents to the clinic with dizziness. Her rhythm strip is shown above. What is the most likely diagnosis? A) First degree atrioventricular block B) Mobitz type 1 second degree atrioventricular block C) Mobitz type 2 second degree atrioventricular block D) Third degree atrioventricular block

C) Mobitz type 2 second degree atrioventricular block

What causes the parotid swelling seen in some patients diagnosed with bulimia nervosa? A) Bacterial infection B) Increased susceptibility to mumps virus C) Noninflammatory stimulation of the salivary glands D) Salivary duct calculi

C) Noninflammatory stimulation of the salivary glands

A 13-year-old male presents to the urgent care clinic with left eyelid swelling, erythema and eye pain which is worse with movement of the eye. He also complains of fever and double vision in his left eye. Which of the following is the most likely diagnosis? A) Bacterial conjunctivitis B) Hordeolum C) Orbital cellulitis D) Preseptal cellulitis

C) Orbital cellulitis

A 74-year-old man with family history of macular degeneration presents with worsening day and night vision and difficulty reading even with his bifocal glasses on. Which of the following fundoscopic findings is more suggestive of exudative advanced macular degeneration? A) A cup-to-disc ratio of 0.3 with arteries narrower than veins B) Accumulation of small yellow deposits on the macula C) Presence of choroidal neovascularization with subretinal fluid and vessel hemorrhages D) Presence of cotton wool spots and flame hemorrhages

C) Presence of choroidal neovascularization with subretinal fluid and vessel hemorrhages A) = no eye disease B) = nonexudative AMD D) = HTN or DM retinopathy

A 38-year-old woman presents with recent onset of new joint pain. Physical exam reveals red, tender, swollen metacarpophalangeal joints of both hands. Lab work reveals elevated ESR and CRP. What is the most likely diagnosis? A) Gout B) Osteoarthritis C) Rheumatoid arthritis D) Temporal arteritis

C) Rheumatoid arthritis

Which of the following antiepileptic drugs has a black box warning concerning teratogenicity? A) Lamotrigine B) Levetiracetam C) Valproic acid D) Zonisamide

C) Valproic acid

Diagnostics of Cholelithiasis

CBC (r/o infection) Amylase and lipase (r/o pancreatic dz) LFTs- elevated alk phos and/or conj bilirubin US- "peas in a pod" HIDA (inconclusive US with high suspicion)

Imaging for Diverticulitis

CT scan (DOC) US MRI X-ray

A 36-year-old man presents with nasal stuffiness, headache, fatigue, facial pain, and chronic post-nasal drip. He has had similar episodes in the past; on average, they have occurred 2 - 3 times a year for the last several years. He has been diagnosed with acute sinusitis, and antibiotics have been prescribed; they have provided him with relief for a brief period. This time, however, his symptoms have bothered him on and off for the last 3 months; he was given a 14-day course of antibiotics, but he experienced only partial relief. He is tired of the recurrent episodes, and he wants a cure. On exam, he is afebrile; nasal mucosa is inflamed, and there is mucopurulent secretion in the nasal cavity. The right maxillary sinus is tender on palpation. Lungs are clear. What is the best next step in the management of this patient?

CT scan of the sinuses

A 56-year-old man presents with moderately severe chest pain. His pain is substernal and left anterolateral; there is some exacerbation on inspiration, and it has been increasing in severity over the last 36 hours. He works as a truck driver and has a history of heavy cigarette smoking, hypertension, and obesity. Over the past week, he has experienced swelling and discomfort in his right calf. Examination shows: BP - 90/55 mm Hg, P - 122/min, RR - 40/min, and temp - 37.6° C. He is mildly agitated and confused. Systemic examination reveals: heart: tachycardia, soft systolic murmur, questionable ventricular gallop. Chest: dullness to percussion left base with scattered crackles and wheezes throughout. Abdomen: negative. Extremities: right calf is 0.5 cm larger than left with some deep tenderness and a trace of ankle edema. Neurologic: no deficits. Laboratory: Hg- 16.4g/dL, Hct 51%, WBC 12,300 cells/ µL, PaO2 52 mmHg, PaCO2 38 mmHg, and pH 7.35. Chest radiograph: borderline cardiomegaly and a prominent aorta; scattered patchy infiltrates bilaterally, small left pleural effusion. What is the most accurate diagnostic modality for diagnosing this patient's condition?

CTA (DOC for PE)

A 36-year-old woman presents with vaginal discharge. She has a history of itching and white discharge. She is sexually active with her husband only. She takes oral contraceptive pills for contraception, and she has never been diagnosed with an STD (sexually transmitted disease).On examination, the vagina is hyperemic and covered with white cottage cheese-appearing discharge. What is the most likely diagnosis?

Candida vaginitis

A family presents in the middle of winter. They live in a low-income housing development. Their gas furnace is broken, and they have been using a kerosene heater at night. For the last 3 days, they have all been experiencing varying degrees of headache, dizziness, nausea, vomiting, and fatigue; symptoms are particularly severe at night. A 4-year-old child has also been very lethargic; occasionally, she seems to black out or fall asleep very soundly. She and her 9-year-old sibling have also had a cough, runny nose and sore throat for the past week. On exam, other than seeming tired, findings are nonspecific. Both children have a runny nose but their lungs and ears are clear. What is the best test to confirm exposure of the most likely diagnosis in this case?

Carboxyhemoglobin level (DOC for CO poisoning)

A 66-year-old man presents with sudden onset of brief episodes of blindness in his right eye, with complete recovery before the next episode. The event is described as a shade coming down across his field of vision, and it is not painful. What is the most common underlying condition leading to the event described above?

Carotid stenosis

Hyperglycemic Hyperosmolar State

Characterized by more severe hypergylcemia but no ketoacidosis

A 35-year-old woman presents with a 24-hour history of fever, right flank pain, vomiting, dysuria, and hematuria. A urinalysis reveals large amounts of red and white blood cells, as well as leukocyte esterase, and a subsequent urine culture was positive for >100,000 Escherichia coli. The patient denies having a history of renal calculi, and a bedside renal ultrasound does not show any stones or hydronephrosis bilaterally. She is diagnosed with a right-sided pyelonephritis and admitted for pain control, hydration, and IV antibiotic therapy. Her symptoms, including her fever, abate quickly, and she is discharged after 48 hours of being afebrile. Which prescription would she most likely receive upon discharge?

Ciprofloxacin 500mg PO BID x 14 days (appropriate choice for more complicated cases with suspected E. coli)

Diagnostics for Asthma

Clinical PFTs (spirometry, bronchodilator response, bronchoprovocation, peak expiratory flow, exhaled NO) Allergy testing Chest x-ray

Labs for B12 Deficiency

Cobalamin level <200 Increased serum MMA Increased homocysteine level

A 35-year-old man has a routine physical examination with no abnormal findings. His family history, however, is positive for familial adenomatous polyposis. What screening test would be best for him to obtain?

Colonoscopy

When to refer for COPD

Complicating co-morbidities Refractory disease Exacerbation

Treatment of DKA

Correction of fluid and electrolyte abnormalities IV insulin Bicarb Monitor glucose every hour and CMP/venous pH every 2-4 hours until stable

Treatment of HHS

Correction of fluid and electrolyte abnormalities IV insulin Monitor Glucose every hour and CMP every 2-4 hours until stable

A 15-year-old boy develops bloody diarrhea with abdominal cramping. Double contrast barium enema shows fine serrations and narrowing of the rectum and sigmoid. Stool contains mucus, blood, and white blood cells, but no parasites or bacterial pathogens. Endoscopy shows inflamed mucosa and pseudopolyps. Biopsy finds extensive inflammatory process in the mucosa and submucosa. Glands are filled with eosinophilic secretions. There is also mild involvement of the terminal ileum. Sulfasalazine treatment was attempted but failed to bring improvement.What is the most appropriate next step in the management?

Corticosteroids While sulfasalazine or 5-amino-salicylacid are first line for UC, severe cases usually need corticosteroids as well.

A 28-year-old woman presents to the office with complaints of generalized fatigue, weight gain, and depression that began a few months ago and has worsened. Upon examination you notice she has some periorbital edema, and upon palpation, a slightly enlarged thyroid. Which test remains the most sensitive screening tool for diagnosing primary hypothyroidism? A) Free thyroxine (T4) B) Free thyroxine index C) Free triiodothyronine (T3) D) Third-generation thyroid-stimulating hormone assay

D) Third-generation thyroid-stimulating hormone assay

Lifestyle modifications for HTN

DASH diet (<2500mg) Potassium supplementation Weight loss Exercise Limit alcohol intake (<1 oz ethanol per day)

Treatment of Moderate Persistent Asthma

Daily inhaled low-dose steroid + LABA or inhaled medium-dose steroid -alternative to LABA: LAMA or leukotriene modifier SABA prn

Treatment of Mild Persistent Asthma

Daily low-dose inhaled steroid or combo steroid-LABA inhaler -alternative to steroid: leukotriene modifier SABA prn

Moderate Persistent Asthma

Daily symptoms Night awakenings once a week Daily SABA use FEV1 >60 and <80% of predicted FEV1/FVC below normal

Intermittent Asthma

Daytime symptoms <2 times a week <2 night awakenings a month SABA to relieve symptoms <2 a week No interference in activity between exacerbations FEV1 and FEV1/FVC normal between exacerbations 0-1 exacerbations requiring steroids per year

SIG E CAPS

Depressed mood Sleep changes Interest loss Guilt/worthlessness Energy lacking Cognition/concentration Appetite changes Psychology Suicide.death preoccupation *screening for depression*

A 54-year-old man presents with chest pain. He has a past medical history of hypertension and diabetes mellitus. The pain is located in the middle of his chest and radiates to his jaw. The pain began about 20 minutes ago, and he rates the pain as a 10 on a 0 - 10 point scale, with 10 being the worst pain he has ever felt. He has had 3 similar episodes, but they have always resolved after 5 minutes or so of rest. He has smoked 1 pack of cigarettes a day for the past 36 years. He drinks 2 or 3 beers on Friday nights. Review of systems (ROS) is positive for diaphoresis, acute dyspnea, and impending doom. ROS negative for fever, chills, and malaise. Physical exam shows an obese, middle-aged man in moderate distress. BP 126/80, pulse 100, respirations 26. Heart and lung exams are normal, except for tachycardia and tachypnea. He has no pedal edema. What aspect of the patient's history is the largest risk factor for an acute myocardial infarction?

Diabetes Mellitus -independent risk factor for atherosclerosis -risk of MI in pt with DM = risk in pt w/o DM but with hx of MI

A 66-year-old man presents to the office with polyuria and erectile dysfunction. He denies any other symptoms or significant past medical history. Physical examination reveals Tanner stage 5 of the external genitalia, balanitis of an uncircumcised penis, and slightly enlarged, symmetrical and smooth prostate. His condition is most likely the result of:

Diabetes mellitus

When to refer for asthma

Diagnosis is uncertain Difficult to control/frequent exacerbations Medications S/E are intolerable

Lifestyle treatment for DM

Diet Exercise +/- psychological intervention Self-monitoring

A 50-year-old man presents with a lump in his groin for 2 weeks. He states he was lifting an air conditioner and he felt a pop in his groin and began to notice an outpouching in his lower abdomen that has become mildly tender over the last week. Resting and lying flat appears to help, and standing and lifting aggravates it. He denies any fevers, nausea, vomiting, or changes in bowel habits. Patient denies any previous abdominal surgeries or procedures. Upon examination, you identify a soft, reducible mass in the lower abdomen and hernia examination reveals a mass pushing against the side of your finger. You order an ultrasound of the lower abdomen and find the intestinal sac has traversed through a weakened area of the abdominal wall and through Hesselbach's Triangle. This type of hernia is referred to as a(n)

Direct inguinal hernia

Treatment of Choledocholithiasis

ERCP Treat regardless of symptoms

Labs for Diverticulitis

Elevated CRP Mild luekocytosis +/- elevated pancreatic enzymes UA- sterile pyuria

A 45-year-old man presents with a 2-year history of worsening dyspnea and a 1-month history of a dry cough. The patient gives no history of fever, chills, chest pain, or wheeze. History is significant for smoking (25 cigarettes/day for more than 22 years). A chest X-ray shows hyperinflated lungs with bullae, tubular heart, flattened diaphragm, and no areas of consolidation. Pulmonary function tests (PFT) reveal a decrease in forced expiratory volume in 1 second (FEV1) along with reduction of FEV1/FVC (forced vital capacity) ratio. These findings are characteristic of what condition?

Emphysema -chronic bronchitis would have a productive cough

A 45-year-old man presents with a 24-hour history of severe anal pain and swelling. The pain started after straining at defecation and has worsened over the course of the day. There is no history of fever. Examination of the anal area reveals a swollen ecchymotic mass in the perianal skin, very close to the anal verge. What is the treatment of choice for this condition?

Excision of thrombosed external hemorrhoid

Screening for Osteoporosis

FRAX (screen for risk factors) DEXA (women >65, men with risk factors)

Diagnosis for Hyperlipidemia

Fasting lipid panel and ASCVD risk calculator: ->10% start statin -7.5-10% consider statin

Medications for HTN

First line: HTCZ or ACEI/ARBs Others: BB, CCBs, A-blockers, diuretics

Lifestyle management of Constipation

Fluid Regular bathroom pattern "Toilet training" Increase dietary fiber

A 29-year-old African American man develops dysuria and increased frequency of micturition. In the emergency room, he is found to have a urinary tract infection and is treated with trimethoprim-sulfamethoxazole (Bactrim), as well as recommended to follow up with his primary care physician in the office. Over the next few days, the patient experiences fatigue, fever, jaundice, abdominal and back pain, and dark urine. Blood tests show Hb of 4g/dl, reticulocyte count of 6%, and MCV of 93. Peripheral smear reveals cell fragments, microspherocytes, and blister or bite cells. Heinz bodies are also present. Based on these findings, which clinical entity is most likely?

G6PD deficiency

Reticulocyte count is increased with

Hemolysis or blood loss *marrow is responding to anemia*

Clinical Presentation of COPD

Hx of smoking Dyspnea Chronic cough Sputum production PE depends on severity

Classic Triad of labs for DKA

Hyperglycemia Anion gap metabolic acidosis Ketonemia

Treatment of complicated H. Pylori

IV PPI until oral medication is tolerable Duration: 4-12 weeks

Initial treatment of Cholangitis

IV hydration Broad-spectrum antibiotics Analgesia (IV opioids)

A 55-year-old African-American man presents as febrile with massive swelling of the abdomen and diarrhea. He has a 20-year history of heavy alcohol use. A fluid wave is elicited on physical examination of the abdomen by striking one flank and feeling the transmitted wave on the opposite flank. In what case would ascitic fluid analysis suggest cirrhosis as a cause of ascites in this patient?

If the fluid has a protein concentration below 3 g/dL. Protein >3: exudates Protein <3: transudates (CHF, cirrhosis, constrictive pericarditis, inferior vena cava obstruction, hypoalbuminemia, Meigs syndrome, some nephrotic syndrome)

Diagnostics of PBC

LFTs- elevated alk phos + AMA Elevated IgM Increased cholesterol Normal imaging Liver biopsy

Diagnostics of PSC

LFTs- elevated alk phos, bilirubin, and transaminases (mild) + PANCA MRCP/ERCP Liver biopsy

Diagnostics for COPD

Labs (CBC, elevated bicarb, AAT deficiency) PFTs (spirometry, peak expiratory flow, lung volume, diffusing capacity) Pulse ox ABG Chest x-ray Chest CT (better test)

Treatment of Constipation

Laxatives (bulk formin, osmotic) Enemas Linaclotide or Lubiprostone

Initial Treatment of GERD without alarm symptoms

Lifestyle/dietary modifications Low dose H2RA

A 55-year-old woman with a 15-year history of type II diabetes presents for follow-up of her diabetes. Her spot albumin/creatinine ratio was 100 mg/g 4 months ago and was confirmed at 100 mg/g yesterday. Her urinary analysis shows no cells, casts, or blood. Her creatinine is 0.7mg/dl and her estimated glomerular filtration rate is 95 ml/min/1.73m2. What medication(s) should you prescribe to help prevent her progression from micro to macroalbuminuria and to help prevent a progressive decline in glomerular filtration rate?

Lisinopril

A 50-year-old man presents to your office for a follow-up appointment of his hypertension. He has complaints of some non-specific chest discomfort, so you decide to perform an ECG. The ECG demonstrates peaked T waves in several leads without any other abnormality. Which of the following medications is most likely to cause this ECG finding?

Lisinopril (due to hyperkalemia)

A 25-year-old man presents with increased urination and thirst. Over the past 3 days, he has been unable to satisfy his thirst and has to urinate up to 20 times per day. He noticed that his urine is very clear and colorless. In general, he feels very weak. He has never had any problems with urination before. His medical history is remarkable for a recently diagnosed psychiatric condition for which he began medical treatment. On physical exam, he appears to be lethargic, dehydrated, and pale. His vital signs are as follows: blood pressure 96/52 mm of Hg, temperature 101.2 degrees Fahrenheit, pulse 108 beats per minute, and a respiratory rate of 26 per minute. Which of the following is a common cause of diabetes insipidus?

Lithium

A previously healthy 35-year-old man has had right flank pain radiating to his right groin for the past 3 hours. CT shows a 7 mm stone in his right ureter. What is the best option of treatment in this patient?

Lithotripsy

A 75-year-old man presents with a 4-month history of dyspnea on exertion and a productive cough. He also unintentionally lost 10 pounds in 2 months. His past medical history is significant for coronary artery disease and a myocardial infarction. He has smoked an occasional cigar the last few years. He has been retired for 12 years, but for 30 years he worked odd jobs in the construction industry. He also helped his father in the family's car garage shop. Vital signs are normal. His physical exam is remarkable for decreased breath sounds in left lower lung fields and dullness to percussion. A chest radiograph is ordered, and it shows a left sided pleural effusion. What condition does this patient have?

Malignant mesothelima -risk factor: work + timing

Treatment of Severe Persistent Asthma

Medium- or high-dose inhaled steroid + LABA +/- leukotriene modifier, Tiotropium, biologic agent

Who to screen for Hyperlipidemia

Men >35 or 20-35 with increased risk of CHD Women >45 or 20-45 with increased risk of CHD Baseline for adults not screened as child Stop at 79 or 65 with >1 normal lipid panel

A 76-year-old woman presents because her children are concerned that that she might have dementia. She states that she is doing reasonably well, except that she sometimes sleeps less deeply and wakes up more often than she did several years ago. According to her children, she is slower than before, and her memory has been getting worse over the last 3 years; she has difficulties recalling the specific date of an event (although she can describe the event itself). She also has a great deal of trouble with names, but she can easily recognize people. She always says: "It is on the tip of my tongue, but..." Aside from hypertension that is under control, she does not have any other health problems. She has been a widow for about 10 years. Her older brother was diagnosed with dementia. Physical examination today is within normal limits for the age, and neurological examination is nonfocal. What should be your next diagnostic step?

Mini-mental status examination

Prevention of PUD

NSAID avoidance H. Pylori irradication

A 52-year-old woman living a non-sedentary lifestyle presents with a 5-day history of low-grade fever, flu-like syndrome, sore throat, and malaise. She is mostly bothered by the fact that she has to "catch" her breath because of pain on inspiration and when coughing. She has no known past medical or surgical history; she is not on any medication, and she has no pertinent family history. She denies any medication use, including over-the-counter medicines. On physical examination, her vitals are: Temperature 100.6 F, pulse 86/min, BP133/75 mm Hg, and RR 20cycles/min. She has shallow breathing, resonant percussion notes, fair air entry with vesicular breath sounds, and friction rub. Her blood gas on room air is: pH 7.36, PCO2 44 mmHg, PO2 100 mmHg, HCO3 26 mEq\L, O2 saturation 99.8%. Her chest X-ray (CXR) is normal and the D-dimer assay is also normal. What is the most appropriate management modality for this patient?

NSAIDs (TOC for pleuritis without effusion)

A 48-year-old Caucasian man presents with severe epigastric pain radiating to the back after a bout of drinking.Two weeks later, a repeat ultrasound showed a round, thin-walled hypoechoic lesion near the pancreas tail measuring 4 cm in its largest diameter and with some calcifications in its walls. MRCP visualized a communication between this cavity and the pancreatic duct. Endoscopic ultrasound detected an area of high-speed flow close to the gastric wall. Which of the following is the most appropriate next step in management?

Observation only (most pancreatic pseudocysts spontaneously resolve)

Clinical Presentation of Cholelithiasis

Occurs with obstruction Postprandial RUQ or epigastric pain (+/- radiation to right shoulder) Nausea/vomiting Afebrile Asymptomatic (80%)

Medications for Osteoporosis

Oral Bisphosphonates (Alendronate or Risedronate) Severe: anabolic agent

A 67-year-old woman presents saying her husband says she doesn't listen to anything he says. The patient states that occasionally she has to ask people to repeat themselves when sitting to her right. She denies any dizziness, headaches, or visual disturbances. Her current medication is furosemide. On physical examination, the Weber test reveals lateralization to the left ear. Air conduction lasted for 15 seconds and bone conduction lasted 10 seconds. What do you suspect as the cause of this hearing loss?

Ototoxicity (secondary to furosemide)

Additional management options for COPD

PDE-4 inhibitors Chronic abx Systemic steroids Oxygen Surgery Palliative

Treatment of PUD

PPI over H2RA Duration: based on ulcer and cause

Treatment of uncomplicated H. Pylori

PPI, Amoxicillin, and Clindamycin for 14 days

Evaluation of Constipation

Palpation of abdomen CMP, CBC, TFTs Imaging (CT, MRI, x-ray)

A 46-year-old woman presents with a 2-month history of heartburn, epigastric discomfort, nausea, and occasional vomiting. She has a history of hyperlipidemia, controlled with diet and exercise, as well as asthma, for which she takes inhalers as needed. She takes no other medications, including over-the-counter analgesics. Family history is noncontributory. On exam she is afebrile, BP120/70 mm Hg, pulse 74/min, and SPO2 92%. Lungs are clear, and she has minimal epigastric tenderness. Otherwise, physical examination is unremarkable. She is advised by her physician to take lansoprazole once daily, which provides only partial relief. Endoscopy is then recommended, which shows a duodenal ulcer. Biopsy reveals infection with Helicobacter pylori. What would be the recommended regimen at this time?

Pantroprazole, amoxicillin, and clarithromycin twice daily for 2 weeks

A 54-year-old woman presents with nervousness, a tremor, and irritability. She indicates that she has lost some weight over the last few weeks even though her appetite has increased. She also says that she is feeling feverish. You suspect that she may have a thyroid condition and order the appropriate labs. The lab results are as follows: TSH 7.8 (0.5 - 5.5 mU/L) T4 4.2 (0.8 - 1.8 ng/dl) T3 5.3 (1.4 - 4.2 pg/ml) What is the most likely diagnosis?

Pituitary adenoma All elevated: think higher in the HPT axis

During an annual physical exam, your patient states that although she does not currently have an outbreak, she has had 7 outbreaks of genital herpes this year. She asks you how she can prevent future outbreaks. As per CDC recommendations, what action will you take?

Place her on suppressive therapy (>6 outbreaks a year)

A 35-year-old man is admitted to the hospital with progressive shortness of breath, fever, and worsening cough. The patient had been in good health until 2 months ago, when he began losing weight. This was associated with anorexia, intermittent diarrhea, night sweats, and then a nonproductive cough. He had lost more than 20 pounds by the time he was admitted to the hospital. His past medical history is unremarkable. He has been divorced for 5 years, and he has 1 child. He is employed as a medical equipment salesman, traveling extensively in the Midwest. He admits to drinking alcohol in large amounts on weekends, but he denies tobacco and intravenous drug use. He gives history of a previous homosexual encounter. Physical examination shows that the chest was normal to percussion and clear by auscultation, except for a few scattered ronchi. The heart is normal except for tachycardia. The abdomen is soft with normal bowel sounds. Genitalia are normal; however, there is a painful 2 cm ulceration at the anal verge. The neurologic exam is unremarkable. Chest radiological findings show diffuse bilateral interstitial infiltrates. Arterial blood gases on room air show pO2- 57mmHg, pCO2 31 mmHg, and pH 7.45. His alveolar-arterial O2 gradient is 55mmHg. Bronchoalveolar lavage fluid with lung biopsy shows the presence of cysts. Sputum cytology is negative for acid-fast bacilli. What is the most likely diagnosis?

Pneuocystis pneumonia

A 5-year-old girl has paroxysms of cough that increase in severity and duration. Some coughing episodes are followed by a whooplike, high-pitched inspiratory noise, and vomiting has also occurred after paroxysms. What laboratory test could lead to the earliest confirmation of the likely diagnosis?

Polymerase Chain Reaction assay and antigen detection (most reliable for diagnostic for B. pertussisor parapertussis infection)

Risk Factors for SBO

Prior abdominal surgery (!!!) Hernia Intestinal inflammation Hx of or increased risk of cancer Prior abdominal or pelvic radiation Hx of FB ingestion

An 80-year-old man was treated for ventricular arrhythmias. He presents 1 month later with joint pains. He also has an unusual mask-like rash over his face and body. Discontinuation of drug therapy causes the symptoms to abate. What drug was most likely administered to this patient?

Procainamide (common long-term control medication for arrhythmias)

Charcot's Triad

RUQ pain Fever Jaundice

Clinical Presentation of Cholecystitis

RUQ pain Nausea/vomiting Low-grade fever + Murphy's sign

Clinical Presentation of DKA

Rapid evolution Polyuria Polydipsia Weight loss Hyperventilation N/V, abdominal pain Signs of dehydration Fruity odor Kussmaul respiration

A 35-year-old woman is reluctant to try an antidepressant for fear of becoming a drug addict. How should the physician respond?

Reassure her that antidepressants are not drugs of abuse because of oral administration, lack of immediate reward, and lack of tendency to cause tolerance

The mother of a 3-year-old boy asks to have a blood test done on her son for lead poisoning. He has not been tested before. They have moved into an older home built before 1960. She has noticed some peeling paint on windowsills and doors and has seen small paint chips on the floors. They are now having the house repainted and are staying with relatives. A careful environmental history is obtained, risk reduction and nutrition education is provided. His fingerstick blood lead level comes back at 13mcg/dL. Which additional management should be done at this level?

Repeat capillary samples, confirming with venous sample within 1 month

A 39-year-old man presents with a 1-week history of severe chest pain. He states that the pain seems to worsen when he lies down. He describes the pain as radiating to the back, and it also worsens when he takes a deep breath. His vital signs are as follows: blood pressure 124/ 84 mm Hg, respiratory rate 18/ min, temperature 101°F, and pulse 74 beats per minute. On auscultation of the chest, you cannot distinguish a S1 or S2 but hear a scratching or grating sound. What is the first step in the treatment of this patient?

Rest and NSAIDs (TOC for acute pericarditis)

Treatment for Acute Asthma Exacerbation

SABA Short course of glucocorticosteroids

Treatment for Intermittent Asthma

SABA prn Trigger control (SABA or LABA + steroid before exposure)

A 45-year-old woman with a no significant past medical history presents with a 4-month history of dull, aching heaviness sensation in her proximal right leg. She notes that this sensation is provoked by extended periods of standing and walking, and is relieved when she lies in a recumbent position. Her past medical history is remarkable for pregnancy 4 times, the most recent being approximately 2 years ago. She denies a history of smoking, trauma, injuries, fever, chills, chest pain, shortness of breath, hemoptysis, cough, skin changes and coolness, and peripheral edema. Her physical exam reveals several dilated, tortuous, elongated veins along the medial right thigh, which are especially pronounced upon standing. The remainder of the physical exam is normal. What will be the most appropriate therapeutic approach for this patient at this time?

Sclerotherapy (TOC for varicose veins)

Bile Pathway

Secreted by hepatocytes into the bile canaliculi Flows to the right and left hepatic duct Common hepatic duct Cystic duct to gallbladder (stored) Back out through cystic duct then common bile Into duodenum through sphincter of Oddi

Iron Studies

Serum iron Total iron binding capacity (TIBC) Transferrin Transferrin saturation Ferritin

Clinical Presentation of Pancreatitis from gallstones

Sharp pain -sudden onset -radiation to the back

Mild Persistent Asthma

Symptoms >2 a week 3-4 night awakenings a month SABA for symptoms relief >2 days/week Minor interference with normal activities FEV1 normal

A 15-year-old girl presents with a 1-hour history of rapid heartbeat, faintness, sweating, and nervousness. She is also experiencing shortness of breath and chest pain. The patient has no significant past medical history. There is no history of similar episodes. The patient is on no medications, and she denies illicit drug use. On exam, her vital signs are BP70/60 mmHg; pulse 200 bpm; RR 22/min, temperature afebrile. She looks pale, and her palms are slightly sweaty. She is not comfortable sitting up, so she prefers lying down. She looks slightly apprehensive. Her heart and lung exam are negative except for the tachycardia; except for cool sweaty hands, a brief abdominal and extremity exam are non-revealing. The physician quickly places the paddles on the patient's chest to record the rhythm; this shows a narrow-complex regular tachycardia at 210 bpm. He requests oxygen, IV line, and continuous monitoring. An EKG is in the process of being completed. At this point, what should be done?

Synchronized cardioversion (TOC for hemodynamically unstable SVT)

A 17-year-old male high school football player presents after being tackled and slammed onto his right dominant shoulder forcefully 2 hours ago during a game. He had immediate pain, but was able to continue punting. He has full active and passive range of motion, but some pain (4/10) with abduction. There is no obvious deformity and the skin over the shoulder is intact and not tented. What would you expect to find on a physical exam?

Tenderness over the AC joint (most likely Type 1) -MC MOI: direct blow to lateral shoulder

A 42-year-old gravida 1, para 1 presents with a 4-month history of menorrhagia. She is having shortened menstrual cycles that are sometimes only 15 days in length, with menstrual bleeding for 5 - 6 days. She is using approximately 12 - 14 pads or tampons per day. She admits to fatigue, headaches, and occasional dizziness, but denies syncope. There is no dysmenorrhea. A thyroid-stimulating test last month was within normal limits. Abdominal and pelvic exams are normal. What statement is true regarding dysfunctional uterine bleeding?

The condition is MC after 40 years of age (50% of cases) -also seen in adolescents (20%) with anovulatory cycles

A 16-year-old Caucasian girl presents with a 6-month history of blackheads and whiteheads on her face. On examination, you find a few papules and pustules on her cheeks; there are no nodules. Her mother reports that she also had similar spots on her face when she was a teenager. What is an appropriate first-line medication for her?

Topical tretinoin

Inpatient Management of Diverticulitis

Treat complications IV abx and fluids Pain control Surgery

Initial Treatment of GERD with alarm symptoms

Upper EGD (within 2 weeks) Further evaluation based on results Treatment the same as w/o alarm symptoms after evaluation is complete

Diagnostics for PUD

Upper endoscopy with H. pylori biopsy H. pylori fecal or breath test

Lifestyle management for Hyperlipidemia

Weight loss Exercise Diet low in saturated fats

A 26-year-old woman presents to your office for an infertility consult. She and her husband have been trying to conceive for the past three years but have been unsuccessful. Upon questioning, she says that her menstrual cycles are very irregular, sometimes as long as two to three months may pass between periods. Upon examination, you notice that she has facial acne and is overweight with a BMI of 30. You order a transvaginal ultrasound. What sonographic findings would help to confirm your suspected diagnosis? A) Endometrioma B) Multiple uterine fibroids C) String-of-pearl appearance on the ovaries D) Uterine polyps

C) String-of-pearl appearance on the ovaries

A 27-year-old woman is on an estrogen-progestin oral contraceptive. Which of the following mechanisms of action is the most important for providing contraception? A) Alteration in cervical mucus B) Impairment of normal tubal motility and peristalsis C) Inhibition of the midcycle luteinizing hormone surge D) Rendering the endometrium less suitable for implantation

C) Inhibition of the midcycle luteinizing hormone surge

Which of the following is considered a minor diagnostic criterion for diagnosis of infective endocarditis, according to the Duke criteria? A) Deep vein thrombosis B) Heberden's nodes C) Intravenous drug abuse D) Temperature of 99.0°F

C) Intravenous drug abuse Major criteria: -Positive blood culture (typical x2, persistent, single of Coxiella burnetli) -Evidence of endocardial involvement (echo, new valvular regurgitation) Minor criteria: -IVDA or HA condition -Dever -Vascular phenomena -Microbiologic evidence

A 77-year-old woman presents with dizziness and being light headed. She denies any new medications or other recent changes. Vitals are temperature 98.0°F, blood pressure 93/56 mm Hg, and pulse 72 bpm. Physical exam is unremarkable. Her medications include metoprolol 50 mg daily, atorvastatin 10 mg daily, vitamin D 1,000 IU daily, and acetaminophen 500 mg as needed. Which of the following is the most likely cause of her dizziness? A) Acetaminophen use B) Atorvastatin use C) Metoprolol use D) Vitamin D supplementation

C) Metoprolol use

What are the two most common organs or systems of the body affected by systemic lupus erythematosus? A) Cardiovascular and pulmonary systems B) Hematologic system and kidneys C) Musculoskeletal system and skin D) Nervous and reproductive systems

C) Musculoskeletal system and skin

Which of the following factors causes a urinary tract infection to be considered a complicated infection rather than an uncomplicated infection? A) History of pyelonephritis B) Hypertension C) Pregnancy D) Previous urinary tract infection within the last month

C) Pregnancy

A 65-year-old woman with no known illness presents with dyspnea and a grade 3/6 systolic murmur. Which of the following would suggest the diagnosis of aortic stenosis rather than mitral valve regurgitation? A) Hyperdynamic left ventricle B) Murmur best heard at apex C) Prominent fourth heart sound D) Wide splitting of S2

C) Prominent fourth heart sound Aortic Stenosis = harsh holosystolic murmur best heard at the right sternal border, absent or delayed aortic component of S2 -more calcification = LV works harder = S4

A 54-year-old man presents with urinary frequency and urgency over the past several days. He notes painful urination as well as chills. His temperature on presentation is 38.6°C. Digital rectal exam reveals a warm, edematous, tender prostate. His urine is cloudy. What is the most likely diagnosis? A) Cystitis B) Epididymitis C) Prostatitis D) Urethritis

C) Prostatitis

A 34-year-old man presents to your office. He notes easy fatigability and dyspnea. On auscultation there is a widened S2 split, and a harsh, non-radiating systolic crescendo-decrescendo murmur located at the second intercostal space which is decreased after the patient performs a Valsalva maneuver. The patient also has a right ventricular precordial lift. Which of the following valvular conditions does this patient have? A) Aortic stenosis B) Mitral stenosis C) Pulmonic stenosis D) Tricuspid stenosis

C) Pulmonic stenosis A) = systolic crescendo-decrescendo B) diastolic decrescendo-crescendo best heard at the apex D) mid-diastolic rumble and opening snap

A 37-year-old woman presents with a history of episodic vertigo, right-sided aural fullness, and right-sided tinnitus for the past several months. The episodes typically last 12 hours. Physical exam reveals her ears to be normal. Audiometric evaluation shows a low-frequency sensorineural hearing loss in the right ear. Dix-Hallpike testing is negative. MRI of the brain and internal auditory canals was normal. Which of the following is the most appropriate treatment for the suspected diagnosis? A) Epley maneuver B) Intratympanic gentamicin C) Restricting sodium intake to 2 to 3 g daily D) Triamterene/hydrochlorothiazide

C) Restricting sodium intake to 2 to 3 g daily Initial treatment for Meniere's disease = low salt, caffeine, nicotine, and EtOH -B and D are both options but reserved for severe cases

Which of the following counseling statements should be given regarding tobacco use and smoking cessation? A) Currently, there are more active smokers in the U.S. than former smokers B) Heart disease risk is reduced within six months of smoking cessation C) Smoking increases the risk for infertility D) There is no benefit to smoking cessation over the age of 80

C) Smoking increases the risk for infertility

A 34-year-old man presents to the clinic with rash and fever. He states he felt like he was developing the "flu" and initially had body aches, malaise and fever of 100.1℉. Soon after, he developed a rash on his body that was painful to the touch. He states he was recently switched from levetiracetam to lamotrigine for his seizure disorder. Physical exam reveals diffuse erythematous macules with scattered bullae and vesicles covering less than 10% of the body. You also note painful oral and genital lesions. Nikolsky's sign is present. Based on these findings, what is the most likely diagnosis? A) Erythema multiforme B) Herpes zoster C) Stevens-Johnson syndrome D) Toxic epidermal necrolysis

C) Stevens-Johnson syndrome

Which of the following statements is true concerning patients with cholestatic jaundice? A) They have decreased hepatic concentration of bile salts B) They have decreased serum concentration of bile salts C) They have decreased total bile salt pool size D) They have increased bile salts in enterohepatic recirculation

C) They have decreased total bile salt pool size -bile salts retained in the parenchyma of the liver lead to down-regulation so less are produced

Which of the following laboratory studies should be ordered in a patient being evaluated for hypertriglyceridemia? A) Antinuclear antibody B) Complete blood count C) Thyroid-stimulating hormone D) Troponin I

C) Thyroid-stimulating hormone (+ lipids, FBS, and LFTs)

A 25-year-old woman presents to your office with a complaint of anxiety. She says that for the past year she has had worsening insomnia, restlessness, irritability, and difficulty concentrating. The symptoms are starting to impact her performance at work so she is interested in treatment. Which of the following is the most appropriate medication? A) Lorazepam B) Nortriptyline C) Quetiapine D) Sertraline

D) Sertraline

Which of the following statements made by a 25-year-old man with lower back pain would warrant a referral to a rheumatologist? A) "My pain gets worse with movement and improves with rest." B) "My pain gets worse with rest and improves with movement." C) "My pain improves as I lean forward on a shopping cart." D) "My pain is worse as the day goes along."

B) "My pain gets worse with rest and improves with movement." A) = benign low back pain C) = stenosis D) = strain or benign

A 47-year-old woman presents with complaints of abdominal pain radiating to her right shoulder, nausea, and vomiting that started seven hours ago. She had spent the day at the state fair where she ate pizza and cheddar fries. Her temperature on presentation is 38.6°C. On physical exam, you palpate the right upper quadrant when the patient inspires and it causes her to stop breathing and wince for a second. What is the most likely diagnosis? A) Acute appendicitis B) Acute cholecystitis C) Acute pancreatitis D) Peptic ulcer disease

B) Acute cholecystitis

A 62-year-old, otherwise healthy, non-smoking woman presents to the clinic with a cough productive of green sputum for seven days. There is low-grade fever and scattered rhonchi on exam, with an oxygen saturation of 97% on room air. Which of the following is the most likely diagnosis? A) Acute bronchitis B) Acute exacerbation of COPD C) Pneumonia D) Upper respiratory infection

A) Acute bronchitis

A 72-year-old-man presents to your clinic with substernal chest pain, shortness of breath and diaphoresis for the last 30 minutes. He states that this started suddenly when he was taking out the trash. He finds minimal relief with rest and worsening pain with even minimal exertion. His past medical history is significant for hypertension, dyslipidemia, and a 40-pack-year smoking history. He denies any history of cardiac disease, recent travel or surgery. Initial chest X-ray shows hyperexpansion of the lungs but is otherwise within normal limits. Electrocardiogram shows normal sinus rhythm without any acute changes. Laboratory analysis shows a troponin 0.03 ng/mL, D-dimer 410 ng/mL and a normal white blood cell count. He feels this is most likely due to heartburn. Given this information, what is the most likely diagnosis? A) Acute coronary syndrome B) Boerhaave syndrome C) Gastroesophageal reflux disease D) Pulmonary embolism

A) Acute coronary syndrome

A 72-year-old man presents complaining of a cough, chest pain, and shortness of breath for the past couple of weeks. He was not concerned enough to seek care until he started noticing bright red blood in his sputum the past couple of days. His weight is down from the last visit. Electrocardiogram is normal. As you await the results of his chest X-ray, what is the most likely diagnosis? A) Adenocarcinoma B) Large cell carcinoma C) Small cell carcinoma D) Squamous cell carcinoma

A) Adenocarcinoma (MC, peripherally located) LCC: less common, wheezing and recurrent pneumonia SCLC: central, 15-20% SCC: 2nd MC, central origin

A 74-year-old man presents with complaints of sudden onset of painful swelling anterior to his left ear and fever. He was recently hospitalized for a total hip replacement. On exam, the patient has tenderness to palpation of his left parotid gland as well as trismus. Purulence is expressed from the left Stensen's duct. What is the treatment of choice for your suspected diagnosis? A) Admission to the hospital for hydration and intravenous antistaphylococcal antibiotics B) Conservative therapy including hydration, parotid massage, and sialogogues C) Incision and drainage D) Outpatient treatment with oral clindamycin

A) Admission to the hospital for hydration and intravenous antistaphylococcal antibiotics -conservative therapy should be used with (never monotherapy) I&D: nonresponsive Outpatient abx: no as life-threatening if it spreads to deep fascial spaces

Which of the following risk factors are the three biggest predictors of future fractures in a patient with osteoporosis? A) Age, low bone mineral density, and history of a previous fracture B) Excess alcohol intake, female sex, and rheumatoid arthritis C) Low bone mineral density, smoking, and family history of osteoporosis D) Use of systemic corticosteroids, age, and smoking

A) Age, low bone mineral density, and history of a previous fracture

A previously healthy 20-year-old woman presents to your office with a complaint of hair loss. Physical exam reveals one smooth, circular patch of hair loss about the size of a quarter. The few remaining hairs in the affected area are noted to have tapering at the proximal end of the hair shaft. A pull test of hair at the edge of the patch is positive. Which of the following is the most likely diagnosis? A) Alopecia areata B) Androgenetic alopecia C) Telogen effluvium D) Tinea capitis

A) Alopecia areata B) = male pattern baldness C) = diffuse 2-3 months after event (birth, stress, diet, meds) D) = areas with pruritus and scaling

A 45-year-old man presents complaining of severe pain on defecation and bright red blood on the toilet paper after wiping for the past couple of days. He denies any other related complaints and states that he only has the pain when defecating and does not notice bleeding at any other time or in his stool. On rectal exam, a linear lesion is seen at the posterior midline area of the anus. What is the most likely diagnosis? A) Anal fissure B) Colon cancer C) External hemorrhoid D) Internal hemorrhoid

A) Anal fissure

Which class of medications is considered first-line therapy in a hypertensive patient with diabetes and proteinuria? A) Angiotensin-converting enzyme inhibitors B) Beta-blockers C) Calcium channel blockers D) Thiazide diuretics

A) Angiotensin-converting enzyme inhibitors

Current guidelines recommend which of the following as long-term follow up of female patients with Hodgkin lymphoma who have been treated to remission? A) Annual mammography starting at age 40 or five to eight years after radiation therapy B) CT scan of chest every two years for the first ten years C) Full body PET scan every two years for the first ten years D) Serum parathyroid hormone level every year for the first five years

A) Annual mammography starting at age 40 or five to eight years after radiation therapy 5 years post: CBC, LDH, Lipid panel, ESR, BS every 3 mos; TSH every year 5-10 years post: CBC, CMP, TSH, FBS every year; Lipid every 6 mos 10 years: CV stress test, echo, carotid US

A 35-year-old woman presents complaining of a recurrent mouth sore. The sore typically last for 5-6 days and then resolves. It usually takes several months before it reoccurs, usually in the same spot on the inside of her left cheek. On exam a whitish ulcer is seen on the oral mucosa, approximately 8 mm in diameter with a reddish halo around it. Rest of exam is normal. What is the most likely diagnosis? A) Aphthous ulcer B) Coxsackie virus C) Herpetiform ulcer D) Varicella zoster virus

A) Aphthous ulcer -MC mouth sore -whitish with erythematous halo and yellow base Coxsackie virus = hand, foot, mouth, Herpetiform ulcer = similar but irregular shape and clusters Varicella zoster virus = shingles, rarely in the mouth

What is the most common anatomic location affected by nummular eczema? A) Arms and legs B) Face and scalp C) Genitals D) Hands and feet

A) Arms and legs

A 56-year-old man is brought by ambulance to the ER complaining of severe chest pain. He is responsive but has great difficulty speaking due to his symptoms. Electrocardiogram shows ST segment elevations in the anterior leads and his oxygen saturation is 89%. While the rest of his history is obtained, what medication(s) should be started immediately if they have not been already by the EMTs? A) Aspirin, nitroglycerin, and oxygen B) Clopidogrel and propranolol C) Lisinopril and warfarin D) No medications, patient should be immediately sent to the cardiac catheterization lab

A) Aspirin, nitroglycerin, and oxygen (MONA)

A 52-year-old woman presents with recent recurrent bouts of shortness of breath with wheezing. Physical exam is normal and there are no other current symptoms. Vitals are normal, her body mass index is 36.5, and she only smoked socially when she was in college. She has a history of allergic rhinitis. What is her strongest predisposing factor for asthma? A) Atopy B) Being overweight C) Exposure to chemicals and pollutants D) Smoking

A) Atopy All others are risk factors for asthma, just less so than atopy

Which of the following best describes the physiologic process responsible for the fourth heart sound heard in patients with advanced aortic stenosis? A) Atrial contraction against a noncompliant left ventricle B) Blood striking a dilated left ventricle during diastole C) Delayed closure of the aortic valve in relation to the pulmonic valve D) Turbulent blood flow across a calcified aortic valve

A) Atrial contraction against a noncompliant left ventricle

A 70-year-old man with long-standing hypertension presents with fatigue and dyspnea. His dyspnea worsens on exertion and when lying supine. Which of the following physical exam findings would be consistent with a diagnosis of congestive heart failure? A) Audible fourth heart sound B) Diminished apical impulse C) Low-grade fever D) Pronounced, diffuse wheezing

A) Audible fourth heart sound (due to stiff, noncompliant left ventricle)

A two-year-old boy presents to the Emergency Department with persistent coughing, choking and wheezing that began about an hour ago. His mother states that he was found playing with some of her older son's small plastic toys before the symptoms began. You suspect foreign body aspiration. What test is the most definitive for diagnosis and treatment of this condition? A) Bronchoscopy B) Expiratory posteroanterior chest radiograph C) Fluoroscopy D) Lateral decubitus chest radiograph

A) Bronchoscopy -all others helpful but not definitive

A 20-year-old man presents with complaints of painful urination and penile discharge. He reports having multiple sexual partners. On exam, there is evidence of purulence expressed from the urethra. No genital ulcers are present. Gram stain of the urethral discharge reveals Gram-negative intracellular diplococci. Which of the following is the most appropriate treatment for the suspected diagnosis? A) Ceftriaxone 250 mg intramuscularly and azithromycin 1 gram orally B) Ciprofloxacin 500 mg orally twice daily for ten days C) Doxycycline 100 mg orally twice daily for seven days D) Metronidazole 2 gram single oral dose

A) Ceftriaxone 250 mg intramuscularly and azithromycin 1 gram orally

What is the most common complication of thoracostomy tube placement? A) Chest tube malposition B) Hemorrhage C) Infection D) Reexpansion pulmonary edema

A) Chest tube malposition

A 60-year-old man with a history of significant benign prostatic hypertrophy presents to the clinic complaining of high levels of anxiety with frequent episodes of tachycardia, profuse sweating, feelings of impending doom, nausea, and tingling in his hands and feet. These episodes last about 10 minutes and are brought on by stressful social situations. The patient reports self-medicating for years with excessive amounts of alcohol. Which of the following medications would be safest to initiate in this patient? A) Citalopram B) Diazepam C) Imipramine D) Lorazepam

A) Citalopram -avoid benzos with alcohol use disorder -no TCAs with BPH

A 55-year-old man presents to the office for his annual check-up. He has not had a physical exam in more than 10 years and is asking to get screened for colon cancer since his brother was recently found to have precancerous polyps. Which test is the method of choice in screening for colonic polyps and colorectal cancer? A) Colonoscopy B) Computed tomography colonography C) Fecal occult blood test D) Flexible sigmoidoscopy

A) Colonoscopy

Which of the following screening tools is a definitive test for the diagnosis of colorectal cancer? A) Colonoscopy B) Fecal immunochemical test C) Fecal occult blood test D) Flexible sigmoidoscopy

A) Colonoscopy

A 47-year-old woman presents complaining of new chest pain of sudden onset. An electrocardiogram is normal. On exam, she is tender to palpation between her sternum and ribs. She works as a landscaper. She does not have a history of obesity, hypertension, hyperlipidemia, or diabetes. What is the most likely diagnosis? A) Costochondritis B) Gastroesophageal reflux disease C) Myocardial infarction D) Pleurisy

A) Costochondritis Key words: TTP between sternum and ribs, landscaper

A 46-year-old woman presents to the clinic with redness and decreased vision in her left eye. She states she was playing frisbee with her kids when it hit her directly in the left eye. Her past medical history is significant for unprovoked pulmonary embolism on life-long apixaban therapy. Physical exam reveals bright red blood occupying 50-60% of the anterior chamber. Visual acuity is significantly decreased on the affected side. You also note mild periorbital ecchymosis, but intact and painless extra-ocular movements. What is the most appropriate next step in management? A) Emergent ophthalmology referral B) Eye patch, cycloplegics, and follow-up with ophthalmology in one week C) Hospitalization and bed rest D) Magnetic resonance imaging of the orbit

A) Emergent ophthalmology referral (traumatic hyphema)

Which of the following signs or symptoms is common in candidal vulvovaginitis? A) Extreme vulvar irritation B) Painful ulcer with an erythematous base C) Purulent discharge D) Tender lymphadenopathy

A) Extreme vulvar irritation

A 27-year-old man presents with a diagnosis of stage I seminoma. What is the initial treatment of choice? A) Orchiectomy B) Orchiectomy plus chemotherapy C) Orchiectomy plus combination chemotherapy and radiation therapy D) Orchiectomy plus radiation therapy

A) Orchiectomy (+surveillance)

A 47-year-old woman presents with complaints of hot flashes and sleep disturbance for the past 18 months. For the past nine months, she has had irregular bleeding lasting up to eight days every six to eight weeks. She complains of dyspareunia. Based on history alone, what is the most appropriate diagnosis? A) Perimenopause B) Postmenopause C) Premature ovarian failure D) Premenopause

A) Perimenopause

A 22-year-old man presents to the urgent care clinic with shortness of breath, facial swelling and rash. His mother states he was stung by a wasp 20 minutes ago and almost immediately began to have difficulties speaking and breathing. He has a heart rate of 110 beats per minute, respiratory rate 36 breaths per minute, oxygen saturation 92% on room air and blood pressure 92/46 mm Hg. An intravenous line has been placed. What is the most appropriate first step in your management? A) Administer diphenhydramine 50 mg IV B) Administer epinephrine 0.3 mg IM C) Administer methylprednisolone 125 mg IV push D) Administer normal saline infusion at 1,000 cc/hr

B) Administer epinephrine 0.3 mg IM

A 40-year-old woman presents to the office with complaints of worsening headaches and double vision over the past three months. After reviewing her chart, you notice that she has gained over 40 pounds since her last visit to the office one year ago. Upon physical exam, you note her weight distribution is mainly in the waist, she has increased adipose tissue in the face, proximal muscle weakness and purple striae over the abdomen. She is not currently taking any prescription or over-the-counter medications. What is the most likely cause of her condition? A) Adrenocorticotropic hormone (ACTH) secreting small cell lung cancer B) Benign pituitary adenoma C) Cortisol secreting adrenal tumor D) Prolonged exposure to excess glucocorticoid therapy

B) Benign pituitary adenoma -MC endogenic cause (consider because no prescription or OTC meds)

Which of the following medications, when used as monotherapy, is most likely to induce mania in patients with bipolar I or bipolar II disorder? A) Bupropion B) Citalopram C) Lithium D) Lorazepam

B) Citalopram

A 64-year-old man with a history of diabetes mellitus presents with pain in his right calf while walking. The pain is relieved with rest. He has smoked one pack of cigarettes per day for the past 35 years. Exam reveals a cool extremity with diminished pulses. What is the gold-standard diagnostic study for your suspected diagnosis? A) Bedside ankle-brachial index B) Contrast arteriography C) Duplex ultrasound D) Plain radiograph

B) Contrast arteriography (gold standard for PAD)

A 28-year-old man presents to clinic who was recently diagnosed with gout and started on allopurinol therapy. Which of the following lab values must be closely monitored when treating a patient with allopurinol over time? A) Aspartate aminotransferase and alanine aminotransferase B) Creatinine C) Hemoglobin and hematocrit D) Uric acid

B) Creatinine -Allopurinol increases kidney excretion of uric acid

A 58-year-old man with a 20 pack-year smoking history complains of a chronic, productive cough. Diffuse wheezing is noted on physical exam. Which of the following additional physical exam findings would be expected? A) Decreased anteroposterior chest diameter B) Decreased heart sounds C) Prolonged inspiratory phase of breathing D) Splitting of the first heart sound

B) Decreases breath sounds -increased AP diameter -prolonged expiratory phase -splitting of S2

A previously healthy 21-year-old woman presents to your office with complaints of abdominal pain and diarrhea for the past year. She describes the abdominal pain as located in the lower abdomen and relieved by defecation. Her diarrhea symptoms include a small volume of loose stool, typically after meals. She has tried over the counter antidiarrheal medication without relief. Which of the following is the most appropriate next step in management? A) Colonoscopy B) Dietary modification C) Food allergy testing D) Testing for ova and parasites

B) Dietary modification

A 48-year-old woman with a ten-pack-year history of cigarette smoking presents to the clinic complaining of gnawing epigastric pain for the past three months. The pain is alleviated by ingesting food but worsens three hours after a meal. Physical exam reveals epigastric tenderness with no palpable masses or rebound. What is the most likely diagnosis? A) Angina B) Duodenal ulcer C) Gastric ulcer D) Irritable bowel disease

B) Duodenal ulcer

At what age should the quadrivalent meningococcal vaccine be offered to immunocompetent children? A) Birth B) Eleven years of age C) One year of age D) Two months of age

B) Eleven years of age (2nd at 18) -high risk at 2 and 10 or 19 and 55 Birth: Hep B 2 months: rotavirus, DTaP, Hib, PVC13, inactivated polio 1 year: Varicella and MMR

A two-year-old boy presents late one evening to your ED after an episode of coughing and choking at home after putting an unknown object in his mouth. He is well-appearing and tolerating his own secretions. You obtain the above image (flip to see). Which of the following is the most appropriate next step? A) Admit for removal tomorrow morning B) Emergent subspecialist consult C) Oral challenge with liquids D) Repeat X-ray in four hours for passage

B) Emergent subspecialist consult

Which of the following findings on physical exam may be associated with obstructive sleep apnea? A) Absent tonsils B) Enlarged neck circumference C) Geographic tongue D) Torus palatinus

B) Enlarged neck circumference

A 22-year-old woman presents with a mass in her right breast that she noticed while showering. She has noticed pain in the area just prior to her menstrual cycles each month. An ultrasound is ordered and shows multiple cystic lesions in the right breast. Which of the following historical factors may be contributing to this condition? A) Family history of breast cancer B) Frequent alcohol consumption C) Obesity D) Tobacco use

B) Frequent alcohol consumption

A 46-year-old man presents complaining of discomfort and pruritus around the anus for the past several weeks. The area then became quite painful two days ago. Pain does not seem to get worse during bowel movements but he does notice a small amount of bright red blood on the toilet paper afterward. Physical exam reveals a small soft tissue mass that is just outside the anus with extreme tenderness to palpation. What is the best treatment option at this time? A) Acetaminophen as needed B) Hemorrhoid excision C) Ibuprofen as needed D) Regular warm baths

B) Hemorrhoid excision -change in pain indicates thrombosis

The physician assistant is evaluating a patient suspected of having an iron deficiency anemia. When examining the patient's peripheral blood smear, the diagnosis is reinforced by the presence of which of the following findings? A) Howell-Jolly bodies B) Hypochromic, microcytic red blood cells C) Macrocytic red blood cells D) Schistocytes

B) Hypochromic, microcytic red blood cells Howell-Jolly bodies = asplenia (sickle cell) Macrocyctes = B12 or folate deficiencies Schistocytes = hemolysis

A 67-year-old obese man with a past medical history of heart failure presents with complaints of daytime sleepiness and morning headaches. He notes that he frequently will doze off in church. His wife complains of his snoring. What is the gold-standard diagnostic study for your suspected diagnosis? A) Home sleep apnea testing B) In-lab polysomnography C) Nocturnal electrocardiogram D) Nocturnal pulse oximetry

B) In-lab polysomnography

A previously healthy 35-year-old woman presents to your office with concerns about an upcoming blood test. She is supposed to have her cholesterol levels checked next week and is very anxious about the procedure. She has avoided blood tests in the past out of fear and even gets anxious when she sees someone on television getting an injection or having blood drawn. Which of the following is the most appropriate therapy? A) Haloperidol 30 minutes prior to the procedure B) Lorazepam 30 minutes prior to the procedure C) Referral for dialectical behavior therapy D) Sertraline 30 minutes prior to the procedure

B) Lorazepam 30 minutes prior to the procedure

A 25-year-old man presents to the clinic with generalized fatigue, muscle aches and a rash. He states this has been going on for about three to four days and seems to be getting worse. He was on a canoeing trip in the boundary waters in northern Minnesota three weeks ago, but did not notice any symptoms during that time. He did tick checks every two to three days and never found a tick on his body. Vital signs are within normal limits. Physical exam reveals the rash shown above (flip to see). What is the most likely diagnosis? A) Erythema multiforme B) Lyme disease C) Rocky Mountain spotted fever D) Tinea corporis

B) Lyme disease -"bullseye" lesion + nonspecific symptoms

A 41-year-old woman presents to the clinic with symmetrically distributed, coalescent, hyperpigmented macules in sun-exposed areas of the face. Which of the following is the most likely diagnosis? A) Melanoma B) Melasma C) Seborrheic keratoses D) Solar lentigines

B) Melasma Solar lentigines are similar but begin as a solitary lesion <5 mm in diameter, not symmetrical

Which of the following are typical symptoms of depression? A) Auditory hallucinations B) Multiple somatic complaints C) Narcissism D) Panic attacks

B) Multiple somatic complaints

Which of the following patient complaints is most indicative of angina pectoris? A) Brief, fleeting left-sided chest pain described as a "flutter" B) Occasional substernal chest pain described as a "deep ache" C) Persistent epigastric pain described as "gnawing" D) Sudden, searing, substernal chest pain described as "ripping"

B) Occasional substernal chest pain described as a "deep ache"

Which of the following treatments for chronic obstructive pulmonary disease-related chronic bronchitis is associated with decreased mortality? A) Inhaled glucocorticoids B) Oxygen therapy C) Pneumococcal vaccine D) Short-acting bronchodilators

B) Oxygen Therapy (and smoking cessation) All the other answers improve lung function or minimize further damage/mortality.

A 50-year-old man presents to clinic with worsening right shoulder pain that began four months ago. There was no trauma or inciting event. Over the past month, he has also been experiencing worsening stiffness in the right shoulder that is interfering with his activities of daily living. Clinical exam shows decreased passive and active range of motion of the shoulder in all planes compared to the contralateral shoulder. X-rays of the shoulder are obtained and are normal. Which of the following is the most appropriate initial treatment for this patient? A) Immobilization with a shoulder sling B) Physical therapy with gentle range of motion exercises C) Shoulder arthroscopy with lysis of adhesions D) Shoulder arthroscopy with rotator cuff repair

B) Physical therapy with gentle range of motion exercises

The drug isotretinoin was pulled from the market in 2009 but is still available in limited form. This is a treatment for recalcitrant acne that works by which of the following mechanism of action? A) Killing bacteria B) Reducing the amount of skin oil C) Reducing the effects of hormones D) Unplugging skin pores and preventing them from getting replugged

B) Reducing the amount of skin oil

What is the single most important intervention in the care of someone with chronic obstructive pulmonary disease? A) Frequent utilization of appropriate antibiotics as infections occur B) Smoking cessation C) Strict adherence to pneumococcal and influenza vaccine schedules D) Well structured weight loss program

B) Smoking cessation

Which of the following drugs is indicated for use in smoking cessation? A) Alprazolam B) Amitriptyline C) Bupropion D) Duloxetine

C) Bupropion

Which of the following is appropriate education for a man with newly diagnosed hypertension? A) Alcohol consumption should be limited to 2 ounces of ethanol or less per day B) Brisk walking 20 minutes per day, three days per week will decrease blood pressure C) Dietary intake of potassium (90 meq - 120 meq) may lower both systolic and diastolic blood pressure D) Recommended sodium intake is 2800 mg or less per day

C) Dietary intake of potassium (90 meq - 120 meq) may lower both systolic and diastolic blood pressure -EtOH = <1 oz ethanol -High-intensity for 20 minutes x3 days/week -<2500mg salt

A 32-year-old woman with a body mass index of 35.1 kg/m2 presents to your office with complaints of increased thirst, unintentional weight loss and blurry vision over the past month. Which of the following is the most appropriate diagnostic test? A) Complete blood count B) Fasting C-peptide level C) Fasting plasma glucose D) Stool hemoccult

C) Fasting plasma glucose

A 56-year-old man presents with epigastric abdominal pain that is radiating toward his mid-back that started yesterday. He has associated symptoms of nausea, vomiting, and anorexia. On physical exam, he has marked tenderness in the epigastrium. Labs reveal an elevated lipase. What is the most common etiology of this patient's condition? A) Alcohol use B) Autoimmune disease C) Gallstone-related disease D) Metabolic derangement

C) Gallstone-related disease B-40% A-30% Drugs Hypertriglyceremia/Hypercalcemia Idiopathic Trauma Scorpion sting

A 27-year-old man presents to his primary care provider complaining of intermittent, burning, substernal chest pain for three months. Symptoms are exacerbated by lying flat after meals, drinking coffee and eating fried foods. He also complains of worsening halitosis but denies regurgitation of undigested food, excessive burping, nausea or vomiting. Physical exam is unremarkable. What is the most likely diagnosis? A) Achalasia B) Diffuse esophageal spasm C) Gastroesophageal reflux D) Zenker diverticulum

C) Gastroesophageal reflux

A 24-year-old man presents with complaints of increasing drainage from both eyes associated with mild discomfort for the past day. He denies severe eye pain or blurring of his vision. He also complains of dysuria and urethral discharge. He is sexually active and does not regularly use protection. On examination, his visual acuity is 20/20 in both eyes. There is profuse purulent discharge as well as marked chemosis, lid swelling, and tender preauricular adenopathy. Which of the following is the most appropriate diagnostic evaluation? A) Direct fluorescent antibody testing B) Fluorescent treponemal antibody absorption test C) Gram stain D) Tzanck smear

C) Gram stain -DOC for Neisseria gonorrhoeae A) = varicella zoster infection B) = syphilis D) = Herpes virus

A 45-year-old man presents to clinic for follow up after recently being diagnosed with hepatitis B. Which of the following serologic markers is the best marker of current viral replication and increased infectivity? A) Hepatitis B core antibody B) Hepatitis B envelope antibody C) Hepatitis B envelope antigen D) Hepatitis B surface antibody

C) Hepatitis B envelope antigen A) = past infection B) = chronic w/low infectability D) = immunity

An otherwise healthy 24-year-old man presents with complaints of diarrhea, nausea, vomiting, and crampy abdominal pain. The patient reports ingesting undercooked chicken 24 hours prior. His temperature on presentation is 37.9°C. Vital signs are otherwise unremarkable. Which of the following is the most appropriate treatment for the suspected diagnosis? A) Cefixime 400 mg orally daily for seven days B) Ciprofloxacin 500 mg orally twice daily for seven days C) Hydration D) Trimethoprim-sulfamethoxazole 160 mg/800 mg orally twice daily for seven days

C) Hydration (TOC for Campylobacter)

A 40-year-old man presents for an annual health maintenance check. His physical exam is unremarkable. The following lab values were obtained: white blood cells 8,300/mcL, hemoglobin 16.1 g/dL, hematocrit 45%, platelets 220,000/mcL, high-density lipoprotein 45 mg/dL, low-density lipoprotein 140 mg/dL, triglycerides 148 mg/dL. Which of the following is the most likely diagnosis? A) Anemia of chronic disease B) Chronic myeloid leukemia C) Hyperlipidemia D) Hypertriglyceridemia

C) Hyperlipidemia (LDL and triglycerides are high)

A 35-year-old woman presents to your clinic for a physical exam. She tells you that she had a Papanicolaou test two years ago that was normal. She is wondering if she needs one this year as she has had them frequently in the past. Which of the following cervical cancer screening guidelines is most appropriate to share with the patient? A) Women aged 21-29 years should have cervical cancer and HPV screening every two years B) Women aged 21-65 years should have cervical cancer and HPV screening every two years C) Women aged 30 years and older should have cervical cancer and HPV screening every five years D) Women should have annual screening starting at onset of sexual activity

C) Women aged 30 years and older should have cervical cancer and HPV screening every five years (21-29 every 3 years cytology only)

In which of the following patients is the influenza vaccine contraindicated? A) A 26-year-old pregnant woman B) A 32-year-old woman with a history of Guillain-Barre Syndrome following influenza vaccination C) A 45-year-old man with urticaria following egg exposure D) A 52-year-old man with a history of anaphylaxis after receiving the influenza vaccination

D) A 52-year-old man with a history of anaphylaxis after receiving the influenza vaccination *the others all require precaution*

Which of the following guidelines for breast cancer screening is consistent with those of the United States Preventive Services Task Force (USPSTF)? A) Begin annual screening mammography at age 40 B) Begin annual screening mammography at age 55 C) Continue screening mammography as long as the patient is in good health D) Discontinue screening mammography at age 75

D) Discontinue screening mammography at age 75 -begin at 50 -every 2 years

Which of the following is the most appropriate first-line treatment for symptomatic giardiasis? A) Ciprofloxacin B) Fluconazole C) Ketoconazole D) Metronidazole

D) Metronidazole (only antimicrobial)

A previously healthy 21-year-old woman presents to your clinic with a complaint of pelvic cramping. The cramping generally begins just before her menses, lasts for approximately three days and ends one to two days after her menses begins. Which of the following is the most appropriate therapy? A) Acetaminophen B) Aspirin C) Indomethacin D) Naproxen

D) Naproxen

A 45-year-old man presents with complaints of persistent worsening bilateral heel pain for the past month. He reports that the pain is the worst when he first steps down out of bed in the morning and then it gradually improves but never goes away. On exam, he is the most tender over the sole of the foot near the calcaneus and has a high arch in both feet. The remainder of the exam is unremarkable, except for the fact that he is overweight. What is the most likely diagnosis? A) Achilles tendinitis B) Bone tumor C) Heel spur D) Plantar fasciitis

D) Plantar fasciitis

A 60-year-old man presents for annual physical examination. He notes recent onset of daily headache as well as pruritus after showering. He has splenomegaly on physical exam. Complete blood count is notable for a hemoglobin of 18.7 g/dL and hematocrit of 55%. Further testing reveals JAK2 mutation. What is the most likely diagnosis? A) Chronic myelogenous leukemia B) Folic acid deficiency C) G6PD deficiency D) Polycythemia vera

D) Polycythemia vera

In infectious mononucleosis, where is lymphadenopathy most commonly noted? A) Anterior cervical chain B) Axillary chain C) Inguinal chain D) Posterior cervical chain

D) Posterior cervical chain -can be anterior but posterior is more common for Epstein-Barr

A 62-year-old man presents to the clinic with bilateral eye irritation for the past several months. Physical exam reveals soft, flat, pink, triangular growths extending toward and involving a small portion of the cornea bilaterally. What is the most likely diagnosis? A) Conjunctival neoplasm B) Episcleritis C) Pinguecula D) Pterygium

D) Pterygium

A 44-year-old woman presents to the office with worsening fatigue and dizziness over the past few months. She decided to make an appointment because she had a few near fainting episodes at her indoor office job. After reviewing her chart, you note that she has lost about 15 pounds over the past year. Upon physical exam, you note that she has a blood pressure of 86/62 mm Hg and that she has areas of hyperpigmentation to her face, arms, hands and mucous membranes. Which of the following is most appropriate to diagnose her condition? A) Abdominal CT scan B) Autoantibody test C) Prolactin level D) Rapid adrenocorticotropic hormone (ACTH) stimulation test

D) Rapid adrenocorticotropic hormone (ACTH) stimulation test

What is the most common cause of acute bronchiolitis in pediatric patients? A) Adenovirus B) Influenza virus C) Parainfluenza virus D) Respiratory syncytial virus

D) Respiratory syncytial virus

A 55-year-old man presents to the emergency department with crushing chest pain that began approximately 30 minutes prior to his arrival while he was doing yard work. An electrocardiogram is performed immediately and shown above. The patient is stabilized and transported to the cardiac catheterization lab. Where is the coronary artery blockage most likely to be found in this patient? A) Circumflex artery B) Posterior descending coronary artery C) Proximal left anterior descending coronary artery D) Right coronary artery

D) Right coronary artery Circumflex = lateral (LA and LV) PDC = posterior (not on ECG) LAD = anterior/septal

Which of the following cholesterol-lowering agents has demonstrated regression in coronary atherosclerosis and is indicated as first-line therapy for patients with coronary artery disease? A) Fluvastatin B) Lovastatin C) Niacin D) Rosuvastatin

D) Rosuvastatin

A 17-year-old woman presents for routine evaluation of her asthma. She notes daily symptoms requiring the use of her rescue inhaler as well as nocturnal awakenings two times per week. What is the most appropriate treatment regimen for controlling the patient's asthma symptoms? A) Continue short-acting inhaled beta-2-agonist as needed B) Short-acting inhaled beta-2-agonist as needed plus high-dose inhaled corticosteroid and long-acting inhaled beta-2-agonist C) Short-acting inhaled beta-2-agonist as needed plus low-dose inhaled corticosteroid D) Short-acting inhaled beta-2-agonist as needed plus low-dose inhaled corticosteroid and long-acting inhaled beta-2-agonist

D) Short-acting inhaled beta-2-agonist as needed plus low-dose inhaled corticosteroid and long-acting inhaled beta-2-agonist (TOC for moderate persistent)

An 87-year-old woman presents from the nursing home with swelling of her neck after eating breakfast. She complains of dry mouth. On exam, there is erythema, swelling, and tenderness to her neck in the submandibular region. Pus is expressed through the left Wharton's duct. What is the most likely diagnosis? A) Dental abscess B) Ludwig's angina C) Plunging ranula D) Sialadenitis

D) Sialadenitis -CT or US to r/o abscess or obstruction -Hydration, warm compresses, massage, +/-abx

According to the American Heart Association, which of the following is used as a risk factor for the medical management of atherosclerotic cardiovascular disease? A) Age greater than 35 years B) Blood pressure 120/92 mm Hg C) Sedentary lifestyle D) Total cholesterol 218 mg/dL

D) Total cholesterol 218 mg/dL Other risk factors: >40, male, AA race, current smoke, HDL <40, SBP >130, current HTN meds, DM

A 68-year-old woman presents with worsening right knee pain. She has had pain there for years, but it has gradually increased in the past couple of months. Acetaminophen is no longer effective. Plain films show joint space narrowing bilaterally with osteophytes on the right. Medical history is unremarkable, except for the presence of pyrosis, peptic ulcer disease ten years ago, and lifelong obesity. What treatment would you likely prescribe next? A) Celecoxib 200 mg twice a day B) Hydrocodone 5/325 mg twice a day C) Ibuprofen 800 mg up to four times a day as needed D) Tramadol 50-100 mg up to four times a day as needed

D) Tramadol 50-100 mg up to four times a day as needed

In what part of the prostate does benign prostatic hyperplasia most commonly develop? A) Central zone B) Fibromuscular stroma C) Peripheral zone D) Transitional zone

D) Transitional zone

A 39-year-old HIV-positive homeless man presents with a six week history of cough productive of blood-streaked sputum. He complains of drenching night sweats and significant weight loss since symptom onset. On exam, the patient is febrile and malnourished. Auscultation reveals posttussive rales. Chest X-ray is notable for infiltrates in the bilateral upper lobes.. Sputum culture reveals acid-fast bacilli. What is the most likely diagnosis? A) Lung abscess B) Pneumocystis jirovecii pneumonia C) Sarcoidosis D) Tuberculosis

D) Tuberculosis

What is the initial treatment of choice for patients presenting with acute uncomplicated inflammation of a Meibomian gland? A) Bacitracin ophthalmic ointment B) Incision and curettage C) Oral antibiotics D) Warm compresses

D) Warm compresses (TOC for hordeolum)


Related study sets

Management Chapter 12 - Managing Human Talent

View Set

Comprehensive Federal Government Final Exam Review

View Set

Материки. Австралия.

View Set

Agonists, Antagonists, and Partial Agonists

View Set

4.1 - Central Processing Unit (CPU) Architecture

View Set

Addiction counseling Quiz chapter 1-3

View Set

STUDY UNIT 1: Investigative interviewing

View Set